ORTHOPEDIC MCQS OB 20 BASIC5
ORTHOPEDIC MCQS OB 20 BASIC7
-
FOR ALL MCQS CLICK THE LINK ORTHO MCQ BANK
-
A 25-year-old female presents to the emergency room within increasing left shoulder pain after walking into a door 5 months ago. She previously sustained a femoral fracture 2 years ago after tripping on a rug. Relevant skeletal survey radiographs and tissue biopsy results are shown in Figures A through D. Laboratory investigations show normal glomerular filtration rate and creatinine clearance. Dual energy x-ray absorptiometry (DEXA) scan shows T-score of -1.4 and
-1.2 at the hip and lumbar spine, respectively. Which of the following laboratory values in Figure E most likely reflects this patient's condition?
-
A
-
B
-
C
-
D
-
E
CORRECT ANSWER: 1
This patient has primary hyperparathyroidism. Laboratory investigations are likely to show elevated serum intact parathyroid hormone (PTH), alkaline phosphatase (ALP) and ionized serum calcium, and low serum phosphate.
Primary hyperparathyroidism is most commonly caused by a single adenoma (80-90%). Besides the signs and symptoms of hypercalcemia, patients present
with calcification of menisci and articular cartilage, erosions in hand bones, "salt and pepper skull", and brown tumors (osteoclastomas), which appear as lytic regions expanding the cortex and causing pathological fractures, so named because of hemosiderin deposition.
Singhal et al. reviewed primary hyperparathyroidism. They advocate routine serum calcium levels for patients with pathologic fractures. If this is elevated, total and ionized calcium and intact PTH levels should be obtained. They feel that surgery for orthopaedic stabilization and parathyroidectomy should be performed simultaneously for better outcome.
Mankin et al. reviewed metabolic bone disease. They suggest that patients with mild disease with normal calcium levels do not require treatment. For patients with high calcium levels, treatment should include maintenance of fluid balance, localization and removal of the adenoma, bony stabilization, and medications (calcitonin, estrogen, bisphosphonates, and calcimimetics such as cinacalcet).
Figure A is an AP radiograph showing a lytic expansile lesion with pathological fracture in metadiaphyseal region of left humerus with similar lesion in the
fifth posterior rib. Figure B is an AP radiograph showing a lytic expansile lesion in the third metacarpal of the right hand and the fifth metacarpal of the left hand. Figure C is a low power micrograph of a brown tumor demonstrating a central zone of bone resorption, and filling with fibroblastic tissue, with a peripheral rim of osteoid production. Figure D is a high power micrograph of a brown tumor. In areas of bone resorption, there are numerous osteoclast-like giant cells amidst a fibrous stroma. This is unlike a true giant cell tumor, which lacks a fibrogenic stroma.
Incorrect Answers:
Answer 2: Elevated PTH and ALP, and low serum calcium and high serum phosphate are characteristic of secondary hyperparathyroidism. This occurs in chronic renal disease, where there is overproduction of PTH because of hyperphosphatemia, hypocalcemia,
and impaired 1,25-dihydroxyvitamin D production by the diseased kidneys. This patient has normal renal function. Answer 3: Elevated PTH, ALP, serum calcium and phosphate occur in tertiary hyperparathyroidism. This again occurs in chronic renal disease after prolonged chronic secondary hyperparathyroidism or after renal transplantation, where the parathyroid glands become autonomous and PTH levels do not normalize. This patient has normal renal function.
Answer 4: Normal PTH, low ALP and high serum calcium and phosphate occur in hypophosphatasia. The defect lies in tissue-nonspecific isoenzyme of alkaline phosphatase (TNSALP) and urine phosphoethanolamine levels are
elevated.
Answer 5: Low PTH, normal ALP, low serum calcium and high serum phosphate levels suggest hypoparathyroidism.
-
Which of the following statements is true regarding polymethylmethacrylate (PMMA)?
-
Barium sulfate initiates the polymerization of monomethacrylate
-
It is a ductile material
-
The Young's modulus is between cortical and cancellous bone
-
It resists shear better than compressive forces
-
The polymerization of PMMA is endothermic
CORRECT ANSWER: 3
Young's modulus measures the ability of a material to resist deformation. PMMA has a Young's modulus between cortical and cancellous bone.
PMMA, or bone cement, is frequently used in joint replacement surgery, tumor surgery, and spine surgery. Bone cements are provided as two-component materials, a powder and a liquid. The powder usually contains a polymer, benzoyl peroxide (initiator), and barium sulfate (radio-opacifier). The liquid usually contains a monomer, DMPT (accelerator), and hydroquinone (stabilizer). The two components are mixed and a free radical polymerization occurs when the initiator is mixed with the accelerator.
Webb and Spencer review the current uses and limitations of polymethylmethacrylate in orthopaedic surgery. Amongst other things, they describe PMMA as a brittle, notch- sensitive material with a modulus of elasticity 10x lower than cortical bone and 100x lower than metal stems used as femoral components.
Illustration A shows the components of bone cement. Incorrect Answers:
Answer 1: Benzoyl peroxide is the initiator when the liquid monomer (monomethacrylate) is added to polymer powder (polymethylmethacrylate). Answer 2: PMMA is considered brittle, meaning that it exhibits linear stress stain relationship up until the point of failure.
Answer 4: PMMA resists compression quite well, but handles shear forces poorly. Answer 5: The polymerization of PMMA is exothermic, meaning it gives off heat.
-
With aging, there is a greater loss of mechanical strength in which of the following types of bone?
-
Cortical bone more than trabecular bone
-
Trabecular bone more than cortical bone
-
Cortical bone and trabecular bone equally
-
Only trabecular bone
-
Only cortical bone
CORRECT ANSWER: 2
In the McCalden paper from 1993, tensile testing to failure was done on 235 cortical specimens, while in the McCalden paper from 1997, compressive testing to failure was done on 255 specimens of cancellous bone. The relative change in strength with age was significantly greater for the cancellous bone, which decreased by 8.5 per cent each decade as compared with 5 per cent for the cortical bone. This corresponds with the clinical finding that age-related fractures occur more commonly in areas of cancellous bone. In both cortical and cancellous bone, volume fraction (virtually identical to apparent density) was the most important explanatory variable, accounting for 76 and 92 per cent of the age-related variations in strength, respectively.
-
A study was designed to measure the benefit of subacromial corticosteroid injections. Participants were randomized to methylprednisolone acetate 40 mg with lidocaine 1% or lidocaine 1% alone. The participants were not provided with information of their treatment allocation. The subacromial injection was prepared and
administered by a single orthopaedic surgeon. Results were collected by the orthopaedic surgeon using clinical and patient satisfaction outcome scores at 6, 12, and 24 weeks. Which of the following would best describe this type of study?
-
Double blinded randomized and controlled study
-
Single blinded randomized study
-
Retrospective study
-
Case-control study
-
Prospective cohort study
CORRECT ANSWER: 2
This design of this study would be best described as a single blinded randomized study.
Single-blinded studies are designed to reduce bias in the treatment groups. Treatment allocation and results are withheld from the participants, but the experimenter will be in full possession of the results and outcomes. Experimenters bias can influence the validity of the study and so, a double blinded study in this scenario would have been a better study design.
Cox et al. looked at the design of clinical studies. A triple-blinded randomized study is an extension of the double-blind design where the committee overseeing the response variables and data are not told of the participants allocation within the study. Blinded studies intend to reduce observation bias associated with unblinded studies.
Illustration A shows a single blinded study design. Here there are two interventions, cortisone and no cortisone. The double headed arrow represents the physicians awareness of each treatment. The participants are then randomized (represented by a one way arrow forward) one of the two treatments. They are unaware of which intervention they received.
Incorrect Answers:
Answer 1: Double blinded randomized and controlled studies, the doctor would not be aware of the treatment allocation or groups.
Answer 3: Retrospective studies would look back in time at two groups of patients treated with either methylprednisolone acetate 40 mg with lidocaine
1% or lidocaine 1% alone and compare results.
Answer 4: A case-control trial would look at the effect of adding methylprednisolone acetate 40 mg with lidocaine 1% to a group of participants.
Answer 5: Prospective cohort study would not randomize and blind the patients of their treatment allocation.
-
A 46-year-old bodybuilder experiences the injury shown in Video V. The type of injury shown in this video most commonly occurs in which phase of muscle contraction?
-
The eccentric phase of this isometric exercise
-
The concentric phase of this isometric exercise
-
The eccentric phase of this isokinetic exercise
-
The concentric phase of this isotonic exercise
-
The eccentric phase of this isotonic exercise
CORRECT ANSWER: 5
The patient sustained a right distal biceps rupture while doing a dumbbell biceps curl. Distal biceps ruptures as well as other tendon avulsions most commonly occur during the eccentric phase, although the injury in this video is during the transition from extension to the start of concentric flexion. This is
an isotonic exercise.
In isotonic exercises, the muscle delivers constant force through the range of motion and is at constant tension. The muscle shortens in the concentric phase (elbow flexes in a biceps curl), and lengthens in the eccentric phase (elbow extends). Distal biceps ruptures occur most commonly in the eccentric phase
of loading.
O'Driscoll et al. evaluated the hook test in 45 patients undergoing surgical exploration for distal biceps injuries. They found that sensitivity and specificity was both 100% for the hook test, compared with 92% and 85% respectively, for MRI. They recommend the hook test as a reliable physical exam test for both complete ruptures (abnormal test) and partial ruptures (painful hook test).
The video shows sudden development of a "popeye" deformity during a biceps curl, characteristic of a distal biceps rupture. Illustration A shows the hook
test. The elbow is flexed to 90° and the forearm fully supinated. With an intact biceps tendon, a finger can in inserted under the lateral edge of the biceps tendon between the biceps tendon and underlying brachialis from anterolateral to posteromedial. With a biceps tendon avulsion, there is absence of a cord- like structure (abnormal test).
Incorrect Answers:
Answers 1 and 2: In an isometric exercise, a muscle contracts against with constant length and there is no appreciable joint motion. An example of isometric exercises is pushing against an immovable object. Muscle length and tension remains constant. Answer 3: In an isokinetic exercise (a type of strengthening protocol), muscles contract at a constant velocity, but resistance varies depending on force exerted through the range of motion. Special machines are necessary for this exercise.
Answer 4: Distal biceps ruptures most commonly occur during the eccentric (not concentric) phase.
-
Which of the following medications works through the competitive inhibition of plasminogen activation?
-
Lovenox
-
Rivaroxaban
-
Tranexamic acid
-
Fondaparinux
-
Coumadin
CORRECT ANSWER: 3
Tranexamic acid (TXA) works through the competitive inhibition of plasminogen activation.
TXA (Lysteda) is an antifibrinolytic that promotes and stabilizes clot formation. It competitively inhibits the activation of plasminogen by binding to the lysine binding site. TXA is effective in reducing the need for blood transfusions while not increasing the risk of VTE and renal complications. However, it is still advised that patients with cardiac stents and previous thromboembolic events including ischemic stroke not be administered TXA.
Godier et al. provide a commentary on the mechanism of action of TXA. They discuss how studies have shown decreased rates of myocardial infarction following trauma in patients that received TXA. They believe this is due to
anti-inflammatory effects caused by binding to plasminogen receptors and stunting the fibrinolytic pathway.
Morrison et al. retrospectively review 896 patients who suffered combat injury, with 293 receiving TXA. Despite being more severely injured, the TXA group had a lower rate of mortality than the control group (P=.03).
Illustration A shows the mechanism of action of TXA. In the absence of TXA, plasminogen, tissue plasminogen activator (t-PA) and fibrin form linkages that lead to
fibrinolysis.
Incorrect Answers:
Answer 1: Lovenox is a factor IIa and Xa inhibitor. Answer 2: Rivaroxaban is a direct factor Xa inhibitor. Answer 4: Fondaparinux is an indirect factor Xa inhibitor.
Answer 5: Coumadin works by limiting the production of vitamin-K dependent clotting factors II, VII, IX, and X.
-
A 67-year-old Hispanic female arrives to the emergency department accompanied by her 16-year-old grand-daughter after a fall. During your initial interview, you notice that the patient replies in both Spanish and English. Her sentences are short and incomplete,
and often looks to her grand-daughter prior to answering a question. How should you proceed?
-
Look up your state's legal age requirement and if the grand-daughter is considered legal, then ask to interpret
-
Ask for another, older family member fluent in both Spanish and English to translate
-
Ask the nursing supervisor for a certified live interpreter, and if not available, ask for a certified interpreter telephone service
-
Ask the patient if she is comfortable proceeding in English, and if comfortable proceed
-
Although not certified, continue the interview with your limited Spanish
CORRECT ANSWER: 3
Certified translators (live or via telephone) should be used in any clinical setting in which the patient is not proficient in the physician's language.
Family members, especially minors, should not be utilized as translators. Hispanic patients, when compared to whites, often rely more on family members for advice and decision-making, and more often confide in the treating physician for definitive decision making.
Poon et al. surveyed over 500 Latinos from the California area to identify any trends/barriers in physician-patient interactions. Common themes noted were
shared-decision making and reliance on family as well as allowing the treating physician to drive care, even without full understanding of the clinical scenario.
Katz et al. interviewed 6 focus groups consisting of Hispanics and non-Hispanic whites. Major differences included the Hispanic patients preferring their care driven by physicians, shared decision making with family, and word-of-mouth information gathering. Non- Hispanic whites readily utilized the internet and did not prefer physicians making the decisions, but rather preferred a shared- decision making model.
Gooberman-Hill et al. interviewed and surveyed a small sample of patients and their families during the informed consent process prior to total joint replacement. Overall, these patients desired to participate in a shared-decision making process, but were often too timid to ask questions and wanted to be prompted by the physician.
Incorrect Answers:
Answer 1: Minors or family members should not be utilized as translators. Answer 2: Family members should not be utilized as translators.
Answer 4: Even if a patient states they are comfortable proceeding, if the physician's clinical judgment suggests otherwise, then a certified interpreter should be obtained. Answer 5: Unless certified, you should not conduct any interview in another language.
-
A 25-year-old male sustained the fracture seen in Figure A and undergoes open reduction internal fixation of the injury. What type of plating technique is used for the ulna?
-
Neutralization
-
Locking
-
Compression
-
Bridging
-
Antiglide
CORRECT ANSWER: 4
This is bridge plating as the fracture site is bridged by the plate and fixation is achieved proximal and distal to the zone of the fracture and thus provides relative stability, relative length and alignment. Bridge plating preserves the blood supply to the fracture fragments as the fracture site is undisturbed during the operative procedure. This theoretically makes less dead bone fragments and lets the whole thing heal by secondary bone healing.
-
Which of the following is included in safety checks on the World Health Organization Safe Surgery Guidelines Checklist?
-
Check if antibiotics has been given within 60 minutes prior to skin incision
-
Check if blood products are on standby
-
Check if venous thromboembolism prophylaxis has been ordered
-
Check if the implant representative is either present, or has been notified
-
Check if all members of the surgical and anesthetic team have been properly credentialed
CORRECT ANSWER: 1
The WHO Safe Surgery guidelines checklist requires that antibiotic prophylaxis be given within 60 minutes prior to skin incision. This check is performed between nurse, anesthetist and surgeon.
Preoperative checks are necessary for patient safety. On the WHO checklist, critical preoperative check points for the surgeon include: site marking, patient consent, allergies, blood loss, antibiotic prophylaxis, critical and non-routine steps, case duration, and whether preoperative imaging is readily visible.
The WHO Surgical Safety Checklist (see below) has 3 checkpoints: before induction of anesthesia, before skin incision, and before leaving the operating room.
Haynes et al. prospectively collected data on the outcomes of 3733 patients before, and 3955 patients after introduction of the Surgical Safety Checklist in
8 hospitals in 8 cities. They found an improvement in death rate (1.5% before, 0.8% after) and inpatient complication rate (11.0% before, 7.0% after). They concluded that implementation of the checklist led to reduction in death and complication rates.
Illustration A shows the WHO Surgical Safety Checklist. Incorrect Answers:
Answer 2: The checklist does not check if blood products are on standby.
Answer 3: The checklist does not check if VTE prophylaxis has been ordered. Answer 4: The checklist does not concern implant representatives.
Answer 5: The checklist does not check credentialing of team members
-
A 59-year-old patient undergoing total hip arthroplasty has a genetically inherited thrombophilia disorder. In this disorder, a variant co-factor cannot be inhibited by activated protein C causing overproduction of thrombin. What is this patients clotting disorder?
-
Factor V Leiden
-
Antithrombin III deficiency
-
Familial dysfibrinogenemia
-
Protein S deficiency
-
Congenital deficiency of plasminogen
CORRECT ANSWER: 1
This patient has Factor V Leiden thrombophilia.
Factor V Leiden thrombophilia is a variant of human factor V that causes hypercoagulability. Factor V functions as a cofactor to allow factor Xa to activate thrombin. Thrombin in turn cleaves fibrinogen to form fibrin, which
polymerizes to form a clot. Deficiency prevents efficient inactivation of factor V by activated protein C causing overproduction of thrombin.
Seligsohn and Lubetsky reviewed hereditary thrombophilia. Factor V Leiden is an autosomal dominant genetic condition that exhibits incomplete dominance.
5% of Caucasians in North America have factor V Leiden. It is less common in Latin Americans and African-Americans and rare in people of Asian descent. Illustration A shows the coagulation cascade.
Incorrect Answers:
Answer 2: Antithrombin III (ATIII) is a potent inhibitor of the coagulation cascade. Normally it inhibits coagulation by lysing thrombin and factor Xa. Antithrombin III deficiency causes abnormal clotting.
Answer 3: Familial dysfibrinogenemia is a rare inherited abnormality of fibrin molecules which result in defective fibrin clot formation.
Answer 4: The function of protein S is to inactivate factor Va and factor VIIIa in the clotting cascade. Protein S deficiency causes abnormalities in the clotting cascade. Answer 5: Congenital deficiency of plasminogen is a rare disorder.
Plasminogen is needed to break down clots. Inability will lead to clot formation.
-
BMP is FDA approved for well-defined medical conditions in limited patient populations. In which of the following clinical scenarios is use of rhBMP-2 FDA approved?
-
In a 32-year-old male with an acute, open tibial shaft fracture treated with minimally- invasive locked plating 8 days from the initial injury
-
In the lumbar spine in a 45-year-old female undergoing posterior decompression and instrumented posterolateral fusion from L4 to S1
-
In the cervical spine in a 56-year-old female undergoing anterior cervical discectomy and fusion of C5-C6
-
In the lumbar spine in a 52-year-old male undergoing L5-S1 anterior lumbar interbody fusion for degenerative disc disease and spinal stenosis
-
In the cervical spine in a 60-year-old male undergoing posterior decompression and posterior instrumented fusion of C3-C7
CORRECT ANSWER: 4
rhBMP-2 is FDA approved for use together with the lumbar tapered fusion device (LT Cage; Medtronic) in single-level ALIF from L2 to S1 levels in degenerative disc disease.
rhBMP-2 is also FDA approved for use in open tibial shaft fractures stabilized with an IM nail and treated within 14 days of initial injury. rhBMP-7 has received FDA humanitarian device exemption approval as an alternative to autograft in recalcitrant long bone
nonunions where use of autograft is unfeasible and alternative treatments have failed. It is also approved as an alternative to autograft in compromised patients (with osteoporosis, smoking or diabetes) requiring revision posterolateral/intertransverse lumbar fusion for whom autologous bone and bone marrow harvest are not feasible or are not expected to promote fusion.
Epstein reviewed the indications for rhBMP-2 use and its off-label use. She found that anterior cervical surgery with BMP2 led to dysphagia and airway complications. With posterior lumbar spine surgery, complications included vertebral osteolysis (bone resorption) and ectopic bone formation/heterotopic ossifications, which made revision surgery difficult.
Burkus et al. prospectively compared 46 patients undergoing single-level ALIF with BMP- 2 vs with autograft in an industry sponsored study. They found that patients receiving BMP-2 had higher rates of fusion and improvement in pain and neurologic status at 12 and 24 months compared with autograft, and there were no adverse events. They recommend rhBMP-2 in ALIF procedures to eliminate pain and scarring from iliac crest bone harvest.
Illustration A shows the FDA approved combination of INFUSE (rhBMP-2) and LT-CAGE device.
Incorrect Answers:
Answer 1: rhBMP-2 is approved for open tibial shaft fractures stabilized with
an IM nail and treated within 14 days of the initial injury. It is not approved for use with plates and screws.
Answer 2: rhBMP-2 is not approved for posterior lumbar spine surgery. It is not approved for more than 1 spinal level.
Answers 3 and 5: rhBMP-2 is not approved for cervical spine surgery.
-
A 38-year-old female with a grade IIIB open tibia fracture is scheduled to undergo definitive fixation and subsequent flap coverage with the orthopaedic and plastic surgery teams. She is met in the pre- operative area by the surgical intern
and paperwork is completed per institutional protocol. She is then brought back to the operating room. Which of the following is true regarding the pre-surgical timeout?
-
The surgical intern must be present because he brought the patient to the operating room
-
The timeout cannot begin without the implant representative
-
If both the orthopaedic and plastic surgical teams are present, a single timeout is sufficient for the entire procedure
-
The timeout may be completed as long as the attending is in an adjacent operating room
-
The pre-surgical timeout has not been shown to decrease complication rates
CORRECT ANSWER: 3
If both teams are present, a single timeout is sufficient for the entire procedure. If one team is absent, a second timeout needs to be completed prior to start of the second part of the procedure.
The WHO pre-surgical safety checklist involves assessment at three points during an operative procedure: before induction of anesthesia, before skin incision, and before the patient leaves the operating room. Some components of the checklist include confirming IV access and allergies, administration of antibiotic prophylaxis, and surgical site verification. The surgeon has been shown to be the most effective team member at reducing complications when using the checklist.
Haynes et al. evaluated the effects of the WHO pre-surgical checklist on perioperative complication rates at eight international sites. They found a significant reduction in rates of complications and death in patients over the age of 16 undergoing noncardiac procedures after implementation of the checklist.
Illustration A shows the WHO pre-surgical checklist Incorrect Answers:
Answer 1: The surgical intern will not be involved in the case and does not need to present for the timeout.
Answer 2: The implant representative is not necessary for the timeout. Answer 4: The attending surgeon needs to be present in the same OR for the timeout.
Answer 5: The timeout has been shown to decrease rates of complication and death.
-
Infliximab is a medication associated with opportunistic infections in patients with rheumatoid arthritis. What is the mechanism of action of Infliximab?
-
Inhibition of dihydrofolate reductase (DHFR)
-
Monoclonal antibody against CD20 on B-cell surface
-
Tumor necrosis factor inhibitor
-
Calcineurin inhibitor
-
Glucocorticoid receptor agonist
CORRECT ANSWER: 3
Infliximab is a tumor necrosis factor (TNF) inhibitor.
Infliximab was designed to modify the host immune response in rheumatoid arthritis (RA). It acts to neutralize both extracellular and membrane forms of TNF, a cytokine considered to be of major importance in the pathophysiology of RA. Complications with this medication include increased risk of opportunistic infection and dose-dependent increased risk of malignancies in patients with rheumatoid arthritis.
Bongartz et al. performed a meta-analysis of randomized trials reporting the harmful effects of anti-TNF therapy. For patients treated with anti-TNF antibodies in the included trials, the number needed to harm was 154 (95% CI, 91-500) for 1 additional malignancy within a treatment period of 6 to 12 months. For serious infections, the number needed to harm was 59 (95% CI,
39-125) within a treatment period of 3 to 12 months.
Illustration A shows the biological properties and target cells of TNF. Incorrect Answers:
Answer 1: This is the mechanism of action of Methotrexate. Answer 2: This is the mechanism of action of Rituximab. Answer 4: This is the mechanism of action of Cyclosporin. Answer 5: This is the mechanism of action of Prednisone.
-
A 65-year-old patient who recently underwent abdominal surgery for a diverticular abscess is referred for right knee pain and swelling for 2 days. Physical examination reveals a temperature of
38.3 degrees Celsius and heart rate of 105 bpm. A clinical photograph, results of synovial fluid analysis, and a polarizing microscopy image are seen in Figures A through C. Synovial fluid gram stain and cultures are pending. What is the most appropriate next step in management?
-
Obtain an MRI of the knee
-
Begin allopurinol therapy alone
-
Begin empiric intravenous antibiotics
-
Give an intraarticular steroid injection
-
Obtain a rheumatology consult
CORRECT ANSWER: 3
This patient presents with symptoms and findings highly suspicious for super- imposed septic arthritis and gout. This patient is likely going to need to go to the operating room for emergent I&D (not listed as an answer choice). Because cultures have already been obtained, the most appropriate next step in management would be to begin empiric intravenous antibiotics.
While the incidence of superimposed gout and septic arthritis is low (~1.5%),
it is reported. In the presence of crystals on aspiration, one should not rule out the possibility of septic arthritis. If there are other risk factors for infection, or the synovial WBC count is high (greater than 50,000), then empiric antibiotics should be initiated immediately after cultures are obtained. If the patient has other evidence supporting septic arthritis (positive Gram stain, purulence), emergent I&D is indicated. Even if the initial suspicion is lower, if they have
not respond to antibiotics after 24-48 hours they should undergo I&D.
Shah et al. investigated 265 crystal-positive joint aspirates in a 7-year retrospective study. They found the incidence of concomitant septic arthritis and crystal disease was 1.5% (4 cases including 2 knees, 1 hip, 1 ankle). All 4 patients that had superimposed infection had a WBC of > 50,000. They concluded that the presence of crystals in the synovial fluid cannot
conclusively rule out a septic joint.
Schuind et al. described a case report of concomitant septic arthritis and gout of the wrist. They found that cultures were positive for Staphylococcus aureus while pathological examination revealed aggregated crystals of monosodium urate.
Figure A shows tense, erythematous swelling of the right knee. Figure B is a table showing synovial fluid analysis with elevated WBC and PMN cell count characteristic of septic arthritis. Figure C shows needle-shaped negatively birefringent crystals on polarizing microscopy. Illustration A shows the synovial fluid characteristics in different disease conditions.
Incorrect Answers:
Answer 1: MRI may be performed if the diagnosis is in doubt. However, clinical findings and synovial fluid analysis point to septic arthritis and empiric IV antibiotics should be started.
Answer 2: Allopurinol is indicated as the first line of treatment for chronic gout attack. Answer 4: Intraarticular steroids are appropriate for treatment of gout in the absence of septic arthritis.
Answer 5: While a rheumatology or infectious disease consult might be valuable, it should not be the next step in management.
-
An 82-year-old osteoporotic woman undergoes total hip arthroplasty for osteoarthritis. During implant trialing, a crack is heard. Intraoperative fluoroscopy reveals a long, spiral fracture of the distal femur. The fracture is reduced and fixed with an anatomic locking plate. The rest of the total hip arthroplasty proceeds uneventfully. Following surgery the surgeon has a meeting with the family and apologizes and provides full disclosure, accepts responsibility, provides a detailed explanation as to what happened, and gives reassurance that steps will be taken to prevent recurrences. This communication approach will most likely
-
Lead to lower patient satisfaction rates
-
Lead to higher patient satisfaction rates
-
Lead to higher likelihood the patient will take legal action against surgeon
-
Lead to higher likelihood the patient will change physicians
-
Prevent any legal action
CORRECT ANSWER: 2
The surgeon has provided all the elements of good communication with the patient and family. His actions will lead to improved patient satisfaction, compared to nondisclosure and attempting to push or shirk responsibility.
In the event of a medical error or adverse event, effective communication with the patient and family is necessary. Effective communication should comprise:
an apology, full disclosure (an explanation of what happened), accepting responsibility, and corrective steps that will be taken to prevent recurrence.While accepting responsibility is integral to the explanation process, it is different from accepting blame.
Mazor et al. found patients would more likely change physicians and seek legal advice in situations with a life-threatening outcome where physicians chose
non-disclosure. They recommend full disclosure, acceptance of responsibility, an apology, detailed explanations, and assurances that steps will be taken to prevent recurrences will result in positive outcomes in terms of patient satisfaction, trust, and emotional response.
MacDonald et al. addressed medical errors in an editorial. Besides full disclosure, they feel that an apology is necessary. This includes an acknowledgement of the event and one’s role in the event, and a genuine expression of regret. Apology laws exist to to reduce concerns about legal implications of disclosure and apology.
Incorrect Answers:
Answer 1: This approach will not lead to diminished satisfaction.
Answers 3 and 5: This approach will not lead to increased risk of legal action, but it may not prevent litigation under some circumstances.
Answer 4: Full disclosure may decrease the likelihood of patients changing physicians.
-
The acquired immune response mediated by the interaction between the T cell receptor and major histocompatibility complex requires which of the following first steps?
-
Antigen phagocytosis by T cells
-
Antigen processing and presentation to T cells
-
Antigen recognition by natural killer T cells
-
Antigen proliferation by polymerase chain reaction
-
T cell disulfide bond cleavage and enzymatic processing
CORRECT ANSWER: 2
The process of antigen processing and presentation is the first step of the acquired immune response. This is performed by the so called antigen presentation cells (APC) of which B cells and dentritic cells are two examples. The APC breaks down the protein antigen in a multitude of enzymatic reactions and presents key peptide sequences via the major histocompatibility complex
(MHC) receptors. Once presented on the surface of the APC, the T-cell receptor recognizes the MHC/antigen complex leading to T-cell activation. At no point in this process are antigens phagocytosed by T cells. Natural killer T cells are involved in this sequence, but not for antigen processing and presentation. Polymerase chain reaction is a laboratory tool used to amplify nucleic acid. Finally, disulfide bond cleavage is a step involved in the APCs antigen processing reaction and does not occur in T cells.
-
All of the following are true regarding a physician responding to a formal written complaint by a patient EXCEPT:
-
Complaint should be taken seriously
-
Written response by the physician is required
-
Physician mandated to retain an attorney to address the formal complaint
-
Patient should be supplied with contact information for the institution's patient representative department
-
Law mandates that the physician respond to the formal complaint
CORRECT ANSWER: 3
It is important for the surgeon to know how to manage a formal complaint. An attorney is not required to assist on behalf of the physician when addressing a formal complaint. All complaints must be taken seriously and the physician is required by law to respond to all formal complaints in writing. Verbal
responses to formal, written complaints are not sufficient. If a patient is filing a formal complaint it is advisable that the patient be supplied with contact information for the institution's patient representative department.
-
What laboratory findings would you expect to find in a patient newly diagnosed with renal osteodystrophy?
-
Decreased PTH secretion, hypophosphatemia, and hypocalcemia
-
Increased PTH secretion, hyperphosphatemia, and hypocalcemia
-
Decreased PTH secretion, hypophosphatemia, and hypercalcemia
-
Increased PTH secretion, hyperphosphatemia, and hypercalcemia
-
Hypophosphatemia and hypocalcemia
CORRECT ANSWER: 2
Renal osteodystrophy is characterized by bone mineralization deficiency due to the electrolyte and endocrine abnormalities associated with chronic kidney disease. The pathogenesis of renal osteodystrophy is multifactorial. Hypocalcemia occurs due to the inability of the damaged kidney to convert vitamin D3 into its active form, calcitriol. The hypocalcemia and lack of phosphate excretion by the damaged kidney causes hyperparathyroidism and secondary hyperphosphatemia.
-
Loss of function in the 25(OH) vitamin D1-alpha hydroxylase gene causes which of the following diseases?
-
Hyperphosphatemia
-
Vitamin D resistant rickets
-
Hereditary Vitamin D dependant rickets type I
-
Hereditary Vitamin D dependant rickets type II
-
Hypophosphatemic rickets
CORRECT ANSWER: 3
Loss of function mutations in the 25 (OH) vitamin D hydroxylase gene cause hereditary vitamin D dependant rickets type I. Hereditary vitamin D dependant rickets type II is caused by a defect in intracellular receptor for 1,25-(OH)2- vitamin D3. Both forms of
hereditary vitamin D dependent rickets show decreased serum calcium/phosphorous, elevated alkaline phosphatase/PTH,
but type I has a decrease in the 1,25(OH)2 vitamin D and type II shows a increase in 1,25(OH)2 vitamin D. Vitamin D resistant rickets (Familial Hypophosphatemic Rickets) is caused by a large deletion in the PHEX gene and results in low serum phosphorous, elevated alkaline phosphatase, and normal calcium. Hyperphosphatemia can be caused by mutation in the GALNT3 gene and results in elevated serum phosphate and decreased PTH/calcium.
Kato et al. describe the molecular physiology of vitamin D metabolism and the recognized gene mutations associated with various clinical manifestations of mutations in vitamin D homeostasis.
-
Figure A is a radiograph taken after an open reduction and internal fixation of a periprosthetic distal femur fracture. With this type of hybrid locked plate fixation, what is the difference between screw A and screw B?
-
Screw A can assist in fracture reduction while screw B provides a fixed angle support
-
Screw A provides improved axial stiffness while screw B provides a fixed angle support
-
Screw A can be used to reduce the plate to bone while screw B can be used to lag fracture fragments together
-
Screw A provides a fixed angle support while screw B can be used to reduce the plate to the bone
-
Screw A can be used to lag fracture fragments together and screw B increases the plate bone frictional stability
CORRECT ANSWER: 4
Locking screws provide a fixed angle support and can improve fixation in osteoporotic bone while nonlocking screws can be used to reduce the plate to the bone, lag fracture fragments together and increase the plate bone frictional stability. "Hybrid" locked plate fixation utilizes both screw types in order to assist with difficult fracture fixation such as when there is a short metaphyseal segment and osteoporotic bone.
Zura et al discusses the biomechanical and biological advantages offered by locked plate fixation.
-
Using levels of evidence in research studies, which of the following represents a level II study?
-
Retrospective case control study
-
Prospective cohort study
-
Case report of 3 patients with the same disease
-
High-quality randomized prospective clinical trial
-
The opinion of a review panel at the annual AAOS meeting
CORRECT ANSWER: 2
Level II Evidence includes prospective cohort studies. There are 5 levels of evidence in clinical research. Level I includes randomized clinical trials. Level II includes prospective cohort studies. Level III includes case control studies AND retrospective cohort studies. Level IV includes case series. Level V evidence includes expert opinion, case reports, and personal observation. The cited reference by Bernstein et al is an excellent review of Evidence Based Medicine (EBM) and describes the 5 step process of making medical decisions based on the available, and often limited, evidence. These steps include: 1) Formulate answerable questions. 2) Gather the evidence. 3) Appraise the evidence. 4) Implement the evidence. 5) Evaluate the process. The reference by Spindler et al is another review article that discusses the different levels of evidence for different study types.
-
All of the following are true regarding the assessment of bone mineral density EXCEPT?
-
Bone mineral density loss is not apparent on standard radiographs until 30% to 40% of mineral is lost
-
Dual energy x-ray absorbtiometry (DEXA) is only able to measure mineralization of the axial skeleton
-
Single photon absorbtiometry allows for the analysis of bone mineralization in the appendicular skeleton alone
-
Quantitative computed tomography (QCT) is associated with 10 times the radiation of a DEXA scan
-
MRI allows for accurate discrimination of normal bone from osteopenic bones
without exposing the patient to ionizing radiation
CORRECT ANSWER: 2
Dual energy x-ray absorbtiometry (DEXA) scans allow for accurate assessment of bone mineralization in both the axial and appendicular skeleton.
Over the last decade, DEXA has become a safe, cost-effective and reliable method to quantify bone mineral density. The World Health Organization (WHO) has adopted DEXA derived BMD measurements to define normal bone, osteopenia, and osteoporosis in the adult and paediatric population. The typical DEXA analysis therefore reports a Z-score, which is the number of standard deviations (SDs) that a patient's BMD is above or below the mean value for persons of the patient's age and sex. The T score is the number of SDs the patient's BMD is either above or below the mean value for young patients of the same gender.
Tortolani et al. provide an overview of the techniques used to assess bone mineral density, the pathophysiology of osteopenia, and the evaluation and treatment options for the general pediatric population as well as for patients with specific pediatric disorders. The authors encouraged orthopaedic surgeons to understand the problems of osteopenia in both otherwise healthy children and children with chronic disorders.
Binkovitz & Henwood provide a review of DEXA technique and interpretation with emphasis on the considerations unique to pediatrics. They found that changes in bone size over time, as occurs in normal pediatric development, confound DXA interpretation. This is one of the major limitations in the use of pediatric DXA. However, attempts to correct for or to circumvent this limitation have been proposed and show promise in improving DXA interpretation and
our understanding of bone physiology.
T-score according to the World Health Organization (WHO): A T- score of -1.0 or above is normal bone density.
A T-score between -1.0 and -2.5 means you have low bone density or osteopenia.
A T-score of -2.5 or below is a diagnosis of osteoporosis.
Incorrect Answers:
Answer 1: X-rays are a suboptimal diagnostic tool for osteopenia because bone mineral density loss is not apparent until 40% is already lost.
Answer 3: Although single photon absorbtiometry (SPA) has shown reliable measurements of bone mineral density in the appendicular skeleton, measurements in the spine (axial skeleton) have been unreliable.
Answer 4: QCT is associated with significantly higher radiation compared to DEXA and is therefore less frequently used.
Answer 5: Conventional MRI scans can be used to measure bone mineral density, giving comparable results to quantitative computed tomography (QCT) without the exposure to ionizing radiation.
-
A mother brings her 4-year-old daughter to your clinic for evaluation of knocked knees and short stature. On exam, the patient ambulates with a circumduction gait pattern and frequently falls. Plain radiographs are displayed in Figure A. Laboratory data is obtained and notable for calcium 9.1 mg/dL (RR 8.5-
10.2 mg/dL), phosphorus 2.9 mg/dL (RR 4.3-5.4 mg/dL), alkaline phosphatase 405 U/L (RR 169-
372 U/L), 25-OH vitamin D 38 ng/mL (RR 25-50 ng/mL), 1,25-OH vitamin D 21 pg/mL (RR 24-86 pg/mL), PTH 25 pg/mL (RR 15-65 pg/mL). The child is adopted and family history is not known. Which is
the most likely diagnosis?
-
Physiologic genu valgum
-
Nutritional rickets
-
Familial hypophosphatemic rickets
-
Renal osteodystrophy
-
Osteogenesis imperfecta
CORRECT ANSWER: 3
This patient has genu valgum secondary to familial hypophosphatemic rickets (FHR), an X-linked dominant disorder caused by inability of renal tubules to absorb phosphate resulting in low serum phosphorus, high alkaline phosphatase, low 1,25-OH vitamin D (1,25OHD) in the setting of normal serum calcium, 25-OH vitamin D (25OHD) and PTH.
Rickets is a disorder characterized by defective mineralization of the growth plate and can result from nutritional deficiency, genetic defects in renal phosphate absorption (FHR) or the formation/action of 1,25-OH vitamin D (type I + II vitamin D dependent rickets), or renal dysfunction. Laboratory data can help differentiate the various etiologies. Treatment of FHR includes medical management with calcitriol (1,25-OH vitamin D), as well as surgical correction of lower limb deformities with either hemiepiphysiodesis/guided growth (age < 11-years-old) or corrective osteotomies (age > 11-years-old).
Stevens et al performed a retrospective review of 14 children with rickets and lower extremity deformities treated with hemiepiphysiodesis/guided growth using either staples or 8-plates. The patients demonstrated a gradual correction in the mechanical axis, as well as improvement in the appearance
and width of the hip and ankle physes likely related to normalized gait pattern. The authors therefore recommended early intervention via guided growth to restore and preserve a neutral axis in these patients.
Tortolani et al authored a review article on osteopenia in children, which can result in repeat fractures, pain and limitation of function. Causes of reduced bone mineral density include rickets (nutritional and genetic), osteogenesis imperfecta (OI), juvenile arthritis, osteopenia associated with neuromuscular disorders and idiopathic osteoporosis.
Figure A is a radiograph of the bilateral knees an a skeletally immature patient showing widening of the physeal plates and metaphyseal cupping, which is pathognomonic of rickets. Illustration A is a table highlighting the different laboratory abnormalities seen in the various forms of rickets.
Incorrect Responses:
Answer 1: This is not physiologic genu valgum, as the patient has irregular appearing physes and abnormal laboratory data consistent with rickets. Answer 2: Nutritional rickets is characterized by low calcium, low 25OHD, high
PTH.
Answer 4: Renal osteodystrophy is characterized by low calcium, high phosphorus, low 25OHD, high PTH.
Answer 5: While OI can be a cause of osteopenia and limb deformity, the abnormal laboratory data is more suggestive of rickets.
-
Nitrous oxide has which of the following important effects when used during orthopedic procedures?
-
Potential interaction with oral contrast material
-
Slow induction times
-
Interferes with self-passivation of titanium implants
-
Gaseous abdominal distension
-
Increases risk of metal ion release from stainless steel implants
CORRECT ANSWER: 4
Nitrous oxide is used as an induction agent during anesthesia. Unfortunately, the nitrous oxide is released by the blood into the bowel and is unable to return back to the blood in similar quantities. This gaseous distension is
important during procedures where fluoroscopy of the abdomen is needed, i.e. pelvic and spinal surgery.
The referenced article by Starr et al notes the difficulty of percutaneous pelvic surgery with use of this induction agent and recommends against its use during these procedures. This article also notes that percutaneous reduction and fixation of posterior pelvic ring injuries is safe and has a high level of patient reported satisfaction levels.
-
Which of the following is NOT a component of Virchow's triad?
-
Thrombocytopenia
-
Platelet dysfunction
-
Hypercoagulability
-
Venous stasis
-
Neither thrombocytopenia (answer 1) nor platelet dysfunction (answer 2) are components of Virchow's triad
CORRECT ANSWER: 5
Virchow's Triad of venothrombolic disease is composed of endothelial injury, venous stasis, and hypercoagulability. These factor increase the risk of a deep vein thrombosis (DVT) and can lead to embolic events, such as a pulmonary embolus (PE). Prophylaxis of thromboembolic disease is a controversial issue
in orthopaedic surgery. Currently prophylactic treatment recommendations are based on risk stratification that weighs the risk of thromboembolism vs. bleeding risk. Salvati et al. reviews the recent (2000) advances and recommendations for preventing, diagnosing, and treating thromboemolism. Recently the treatment recommendations have been updated by Haas et al, again weighing the risks of clot vs. bleeding.
-
A mother brings her 4-year-old daughter to you for evaluation of left knee swelling of 3 months duration. The history, exam, and benign radiographs are consistent with a presumptive diagnosis of juvenile idiopathic arthritis. What would
be the most likely result from serum analysis?
-
Normal erythrocyte sedimentation rate (ESR)
-
Normal rheumatoid factor (RF)
-
Normal C-reactive protein (CRP)
-
Elevated Anti-streptolysin O antibody (ASLO)
-
Elevated human leukocyte antigen-B27 (HLA-B27)
CORRECT ANSWER: 2
The patient's presentation is consistent with oligoarticular juvenile idiopathic arthritis. In these patients, serum RF levels are typically normal.
Juvenile idiopathic arthritis (JIA) most commonly presents in females ages 1-3 years old, affecting the knees and ankles most frequently. The diagnosis is one of exclusion, with infection being the most important diagnosis to rule out. The diagnosis is made clinically with onset of symptoms by age 16, pain in any
joint for six or more weeks, and exclusion of other diagnoses.
Punaro reviews the presentation and workup of JIA, reporting that laboratory analysis alone is rarely diagnostic. Typically, non-specific inflammatory markers including ESR and CRP are elevated, and the blood count may demonstrate leukocytosis and anemia.
However, 97% of patients have normal RF levels. HLA-B27 is elevated in only 8% of patients with JIA. Anti-nuclear antibodies (ANA) are so frequently false-positive that the test in isolation is considered to have no diagnostic value.
Gardner et. al. review rheumatologic laboratory interpretation. They recommend that serum ESR, CRP, RF, HLA-B27, ANA, and more be used in conjunction in order to support or rule out certain diagnoses.
Incorrect Answers:
Answer 1: In JIA, ESR is frequently elevated Answer 3: In JIA, CRP is frequently elevated
Answer 4: ASLO is elevated in post-streptocococcal arthropathies
Answer 5: HLA-B27 is considered a risk factor for JIA if present, but is rarely a positive test finding.
-
A 3-year-old boy is brought to the clinic by his mother who is concerned that his legs are bowed. His mother states that he started walking at a normal age but she feels his lower extremity deformity has worsened over the past year. The patient was born in Haiti and was exclusively breastfed. A standing radiograph is shown in Figure A. What associated finding may also be present in this child?
-
Widened wrist physes
-
Elongated hands and fingers
-
Decreased parathyroid hormone
-
Decreased alkaline phosphatase
-
Elevated calcium
CORRECT ANSWER: 1
The patient described most likely has nutritional deficiency rickets. After genu varum, physeal widening is one of the most common findings in nutritional rickets.
Nutritional deficiency rickets is rare in the developed world but may still be seen in dark- skinned patients, those with special dietary preferences, exclusively breastfed children, or children from low-resource countries where dietary supplementation is less common or standardized. These patients have nutritional deficits in Vitamin D and Calcium and are usually found to have low calcium, low phosphate, high alkaline phosphatase, and increased parathyroid hormone (PTH). The disease is characterized by physeal widening and osteopenia on imaging, which may manifest on physical exam with widened wrists and ankles, spinal kyphosis, and genu varum. A rachitic rosary may also be present with thickening of the costochondral junction of the anterior ribs.
Tortolani et al. present a review of bone mineral density deficiencies in pediatric patients, discussing normal physiology of growth as well as pathophysiology of osteogenesis imperfecta, rickets, osteomalacia, juvenile arthritis, and neuromuscular diseases with mineralization disruption. Specific to rickets, they describe findings of genu varum, shortening of long bones, spinal kyphosis, and fractures.
Naik et al. focused on prevention in their randomized controlled trial of maternal vitamin-D supplementation for exclusively breastfed babies. They found that babies of mothers who were given 600,000 IU/day for 10 days after childbirth had significantly higher vitamin-D levels after 6 months than the placebo control group.
Agarwal et al. reviewed a series of over 200 patients with any lower extremity deformity in India and found approximately one quarter to have nutritional rickets. In their series of 53 patients with such rickets, they verify a high incidence of physeal widening (100% distal ulna, 45% radius, 37% fibula,
22% tibia) and discuss eccentric vs diffuse physeal widening.
Figure A is a standing AP bilateral lower extremity radiograph demonstrating significant genu varum, physeal widening, and metaphyseal flaring consistent
with rickets.
Incorrect Answers:
Answer 2: Elongated phalanges are not associated with rickets, but this finding may be seen in Marfan's syndrome.
Answer 3: PTH is increased in nutritional deficiency rickets.
Answer 4: Alkaline phosphatase is increased in nutritional deficiency rickets. Answer 5: Calcium levels are decreased in nutritional deficiency rickets.
-
Which of the following amputations has the LEAST increase in metabolic demand for walking compared to a healthy patient without amputation?
-
Traumatic transtibial amputation
-
Vascular transtibial amputation
-
Vascular thru-knee amputation
-
Traumatic transfemoral amputation
-
Vascular transfemoral amputation
CORRECT ANSWER: 1
Metabolic demands for walking increase after amputation in the lower extremity (Illustration A). In general, traumatic amputations increase the metabolic demands of walking less than those performed for vascular lesions. This is because patients who require amputation for vascular disease likely have continued vascular pathology above the amputation.
Czerniecki reviews normal gait, influence of prosthetic alignment on amputee function, and the effects of prosthetic components on the metabolic costs and the biomechanical function of the amputee.
-
A 58-year-old patient suddenly develops cardiac arrest while undergoing a routine total knee arthoplasty. He is resuscitated with
20% lipid emulsion. What was the most likely causative agent for cardiac arrest?
-
Epinephrine
-
Bisoprolol
-
Bupivacaine
-
Fentanyl
-
Rocuronium
CORRECT ANSWER: 3
This patients cardiac arrest was likely caused by an intravascular bolus of Bupivacaine.
Bupivacaine is a long acting local anesthetic. It acts on intracellular voltage- gated sodium channels to block sodium influx into nerve cells, which prevents depolarization. If administered systemically, bupivacaine can cause serious complications to the cardiovascular system. Side effects include hypotension, arrhythmia, bradycardia, heart block, and cardiac arrest. Treatment should involve urgent administration of intravascular lipid emulsion alongside standard ACLS protocols.
Rosenblatt et al. describe the successful use of a 20% lipid emulsion to resuscitate a patient after a presumed bupivacaine-related cardiac arrest. Resuscitation involved a lipid emulsion bolus of 1 mL/kg given immediately, which was followed by a continuous infusion until the patient stabilized.
Corman et al. evaluated the use of lipid emulsion for reversal of local
anesthetic-induced toxicity. They suggest that lipid emulsion may reverse local anesthetic toxicity by extracting lipophilic local anesthetics from aqueous plasma or tissues or by counteracting local anesthetic inhibition of myocardial fatty acid oxygenation.
Illustration A shows an ECG of a patient with clinical deterioration after systemic administration of bupivicaine. The ECG shows complete heart block with multifocal ventricular beats. The patient progressed to asystole.
Inncorrect Answers:
Answer 1: Intravascular epinephrine would cause hypertension and tachycardia. There is no specific antidote for epinephrine intoxication and treatment is primarily supportive.
Answer 2: Intravascular overdose of bisoprolol would cause hypotension, bradycardia and possible cardiac arrest. Treatment includes treating bradycardia with
atropine, isoproterenol or cardiac pacing. Hypotension should be treated with IV fluids. Continuously monitoring is required.
Answer 4: High doses of fentanyl usually have little significance intra- operatively under the care of the anesthesiologist. The most significant is respiratory depression. Pure opioid antagonists, such as naloxone, are specific antidotes if required.
Answer 5: High doses of Rocuronium will cause respiratory depression, tachycardia and hypertension. Sugammadex (tradename Bridion) is an agent for reversal of neuromuscular blockade by the agent rocuronium in general
anaesthesia.
-
Abnormal duplication of the zone of polarizing activity (ZPA) may lead to which of the following deformities?
-
Figure A
-
Figure B
-
Figure C
-
Figure D
-
Figure E
CORRECT ANSWER: 1
The image in Figure A demonstrates a mirror hand deformity, which is known to be caused by abnormal duplication of the zone of polarizing activity.
During embryologic development, the limb bud has several defined regions. Two important regions are the apical ectodermal ridge (AER) and the zone of polarizing activity (ZPA). The AER controls longitudinal growth. Abnormal developement or loss of the AER will lead to a truncated limb (exact deformity depends on when in development the AER was affected). The ZPA acts via the sonic hedgehog (SHH) pathway to control anterior-posterior and radio-ulnar growth.
Manske et al. provide a review of the embryologic development of the upper extremity and classify various congenital differences. Classifications include failure of formation, failure of differentiation, duplication, overgrowth, undergrowth, constriction band syndrome, and generalized skeletal abnormalities.
Kelikian et al. provides a review of various of congenital differences of the upper exteremity and attempts to classify them based on their developmental origins.
Figure A is photograph of a mirror hand. Illustration A is a diagram of a limb bud with the apical extodermal ridge (AER), showing how it guides longitudinal growth. Illustration B is a diagram of a limb bud showing the ZPA in relation to the AER. Illustration C is a diagram of a hand showing increasing amounts of SHH expression from radial to ulnar. SHH guides growth of the ZPA and the gradient of SHH expression explains how growth is differentiated from radial to ulnar in the developing limb bud.
Incorrect Answers:
Answer 2: Figure B is a photograph of a hand with pre-axial polydactyly (duplicated thumb). Pre-axial polydactyly is sporadic mutation not known to be caused by abnormal development of a specific area of the limb bud. It is often associated with syndromes such as Holt-Oram, Fanconi's anemia, and others. Answer 3: Figure C is a photograph of a hand with post-axial polydactyly (duplicated small finger). Post-axial polydactyly is most commonly seen in
African-Americans who inherit it in an autosomal dominant fashion. In Caucasians, the genetic cause is typically more complex and a full genetic workup is needed.
Answer 4: Figure D is a photograph of a hand with syndactyly.. Syndactyly is a failue of apoptosis that can have varying causes. Autosomal dominant inheritance has been described, along with associations to syndromes such as Poland syndrome, Apert syndrome, and others.
Answer 5: Figure E is a photograph of a hand with a hypoplastic thumb.. The cause of hypoplastic thumb is unclear, but may be associated with numerous
syndromes including VACTERL, Holt-Oram, TAR, and Fanconi's anemia.
-
Bone is biomechanically weakest to resistance of which of the following forces?
-
Tension
-
Compression
-
Inertia
-
Centripetal
-
Shear
CORRECT ANSWER: 5
Bone is weakest in shear and strongest in compression. When a force creates
a tensile stress in a particular region of a loaded bone, failure will occur in that region first. A transverse fracture occurs in a long bone that is subjected to pure bending. The convex portion of the bone is under tension and fails first, the fracture then propagates transversely. A butterfly fragment results from a combination of bending (transverse) and compression (oblique/shear) as the ends of the failing bone are driven together. The production of a butterfly fragment likely depends on the rate and magnitude of the applied load.
-
Which of the following laboratory tools is used to detect genetic translocations for prenatal screening?
-
Western blot analysis
-
Northern blot analysis
-
Enzyme linked immunosorbent assay (ELISA)
-
Cytogenetic analysis
-
Flow cytometry
CORRECT ANSWER: 4
Cytogenetic analysis performed for prenatal testing involves analyzing entire chromosomes, obtained from the nucleus of the cell, to determine changes in chromosomal size and number (e.g. Down Syndrome, trisomy 21). Western blot analysis and ELISA are used to identify absence/presence of specific proteins from a solution.
Northern blot analysis is used to identify absence/presence of specific RNA from a solution. Flow cytometry identifies specific proteins on the cellular surface in order to sort cells with/without this protein from each other. The review article by Bridge reviews specific molecular biological techniques used in orthopaedics for diagnostic purposes.
Rubin et al. reviews the many molecular translocations seen in sarcomas - including ewing’s sarcoma: chromosomes 11 and 22 [t(11;22)]; and synovial
sarcoma: chromosomes X and 18 [t(X;18)]. Illustration A is a visual mnemonic to remember the applications of commonly used tests in molecular biology.
-
Which of the following statements is true regarding articular cartilage?
-
The hydrophilicity of type II collagen molecules gives cartilage its viscoelastic properties
-
Water accounts for approximately 25% of the mass of cartilage
-
Multiple aggrecan molecules link together to form glycosaminoglycans
-
The positive charge formed by aggrecan molecules helps to create the high osmotic swelling pressure of cartilage
-
Type II collagen helps prevent swelling of articular cartilage which would otherwise occur due to high osmotic pressures
CORRECT ANSWER: 5
The one role of collagen in articular cartilage is to provide the structural framework to resist swelling under high osmotic tissue pressures created by aggrecan. Type II collagen is the predominant type in articular cartilage.
Proteoglycans, the most common of which is aggrecan, are produced by chondrocytes and give articular cartilage its hydrophilic properties. Multiple glycosaminoglycans (GAGs), such as chondroitin and keratin can attach to core proteins to form aggrecans. Link proteins then help aggrecans interact with hyaluronic acid. The negative charge of this complex helps create a strong osmotic gradient, which attracts water and increases tissue pressures. Normal aging involves a decrease in the water content of the extracellular matrix while osteoarthritis is associated with increased water content, which leads to loss of strength and elasticity.
Chen et al. evaluated the strain and depth related properties of articular cartilage in bovine models. They found that the zero-strain permeability, zero- strain equilibrium confined compression modulus, and deformation
dependence constant differed among the layers of cartilage. They suggest that the complex strain-dependent properties of articular cartilage of different thickness and location have clinical implications for tissue engineering.
Illustration A is a diagram depicting the extracellular matrix of articular cartilage. Incorrect Answers:
Answer 1: The hydrophilicity of aggrecan molecules contributes to the viscoelastic properties of cartilage.
Answer 2: Water accounts for 65%-80% of cartilage mass Answer 3: Multiple GAGs link to form aggrecan molecules
Answer 4: Aggrecan molecules have a negative charge that contributes to the high osmotic gradient
-
The Chi-square test is most appropriate for which of the following?
-
Comparing means of parametric numeric data between two groups
-
Comparing means of non-parametric numeric data between two groups
-
Comparing means of parametric numeric data between 3 or more groups
-
Comparing categorical data between groups
-
Comparing categorical data between groups with less than 5 samples or data points per group
CORRECT ANSWER: 4
The Chi-square test is most appropriately used for comparing proportions of categorical or ordinal data.
When considering which statistical test to use the investigator must consider how many groups are being compared, and if the data collected is numerical or categorical, and parametric or non-parametric. When the study is comparing proportions of categorical data the appropriate test will be either a
Chi-square test, or a Fisher exact test. The Fisher exact test is preferred when there is less than 5 data points in any group being compared.
Illustration A is a flow chart demonstrating how to determine which statistical test is most
appropriate.
Incorrect answers:
Answer 1: Comparing means of parametric data between 2 groups is done with a t- test or paired t-test
Answer 2: Comparing means of non-parametric data between 2 groups is best done with the Mann-Whitney U test
Answer 3: Comparing means of parametric data between 3 or more groups is best done with Analysis of variance (ANOVA)
Answer 5: Comparing proportions of categorical data between groups when at least one group has less than 5 samples or data points is best done with the
Fisher exact test
-
Integrins function in which of the following ways?
-
Maintain bone and cellular matrix
-
Form bone by producing non-mineralized matrix
-
Form cartilage under intermediate strain and low oxygen tension
-
Attachment of osteoclasts to bone surfaces
-
Binds to RANKL on the osteoblast, preventing RANK activation
CORRECT ANSWER: 4
Integrins are groups of molecules essential for osteoclast attachment to the bone surface.
Integrins are a large family of heteromeric cell surface receptors composed of non- covalently bound alpha and beta subunits which interact with extracellular matrix molecules, serum constituents and various adhesion molecules. Specifically, aVß3 is a type of integrin found on osteoclasts that attaches to bone by coupling with vitronectin.
Bikle reviews the cause of bone loss during periods of unloading. They state the during periods of rest, integrin expression is decreased, leading to disuse osteoporosis. They argue that drugs that selectively regulate the integrin signaling pathway may reverse bone loss during periods of disuse.
Illustration A shows an example of an osteoclast attaching to bone via the integrin aVß3.
Incorrect Answers:
Answer 1: This is the function of osteocytes. Answer 2: This is the function of osteoblasts.
Answer 3: This is the function of osteoprogenitor cells. Answer 5: This is the function of osteoprotegerin.
-
The Arg-Gly-Asp (RGD) sequence of extracellular bone proteins directly allows which of the following?
-
Interaction with integrins
-
Chemoattraction of osteoclasts
-
Binding to vinculin protein
-
Attachment site of Sharpey's fibers
-
Decrease in bone stiffness
CORRECT ANSWER: 1
The Arg-Gly-Asp (RGD) sequence of extracellular bone proteins such as fibronectin and vibronectin allow binding of integrins on the surface of osteoclasts to enable bone resorption.
Bone homeostasis involves molecular regulation that involves osteoblasts, osteoclasts, and bone. Osteoblasts upregulate and down regulate osteoclasts that absorb bone at the ruffled borders. Integrins, on the surface of osteoclasts bind to extracellular proteins
on bone, such as vibronectin, to facilitate bone resorption at the ruffled border. The Arg- Gly-Asp (RGD) sequence plays an important roll in the binding of integrins and extraceullar proteins such as vibronectin and fibronectin. After binding, the intracellular side of these proteins binds to intracellular proteins to allow for change in the intracellular structures and function.
Bosseti et al. review the extracellular matrix interactions involved with bone induction and conduction mechanisms. They note that extracellular binding leads to intracellular protein alteration, which causes different intracellular effects depending on the specific ligand that binds.
Illustration A shows the binding of integrin and vibronectin.
Incorrect Answers:
Answers 2-5: The Arg-Gly-Asp (RGD) sequence does not involve these options.
-
Figure A represents a stress vs. strain curve for a tendon. Which portion of the curve is due to a unique structural characteristic of
elastin?
-
A
-
B
-
C
-
D
-
E
CORRECT ANSWER: 1
The area of the curve labeled A is the "toe region" which represents a change in length before the elastin fibrils within the tendon have fully elongated to the point of stretch.
A stress vs. strain curve for a material, such as a tendon, represents the stress felt across a material compared to how it changes in length (strain). For some materials, such as tendons, there exists a toe region at the beginning of the curve. This region is curved, and then transitions into the elastic region which
is a straight line with a fixed slope. Elastin fibers within the tendon are crimped when at rest. As a force is applied across the tendon these fibers must first expand out until straight before providing significant resistance. Once these fibers are straightened they provide constant resistance
(represented in the elastic region of the curve) until the yield point after which they fail.
Kastelic et al. describe the tendon matrix as wavy at rest and present a "sequential straightening and loading" model for their behavior as an explanation for the toe region of the stress vs. strain curve. They go on to derive mathematical formulas for predicting the degree or curvature for this region depending on the properties of the fibers.
Hansen et al. used optical coherence tomography to measure crimp angles of fibers within a tissue. They measured how these angles change with force applied and found that all fibers were completely straightened at approximately 3% axial strain.
Figure A is an example of a stress vs. strain curve that might be seen for a tendon. Illustration A is the same curve with labels to describe each marked region. Illustration
B is a drawing of the "sequential straightening and lengthening" model proposed by Kastelic et al.
Incorrect answers:
Answer 2: B represents the elastic region of the curve, after all elastin fibrils have completely straightened.
Answer 3: C represents the yield point. Answer 4: D represents the failure region.
Answer 5: E represents the slope of the elastic region, otherwise known as Young's Modulus.
-
Which of the following classes of antibiotics works by binding to the 30S- ribosomal subunit?
-
Aminoglycosides
-
Penicillins
-
Rifampin
-
Macrolides
-
Quinolones
CORRECT ANSWER: 1
Aminoglycosides work by inhibiting peptide elongation by binding to the 30S- ribosomal subunit.
Aminoglycosides are among the oldest classes of antibiotics. They are act by binding to the 30S ribosomal subunit and are considered bactericidal. Due to their effectiveness on Gram-negative bacteria they are often used in conjunction with cephalosporins for treatment of open fractures. Care must be taken when using aminoglycosides due to their potential nephrotoxicity and ototoxicity.
Mader et al. present an instructional course lecture reviewing common antibiotics and their mechanisms of action. For aminoglycosides, they comment that their primary use is for aerobic Gram-negative organisms, particularly enterobacter species and P. aeruginosa. Aminoglycosides have realtively poor activity against Gram-positive organisms and should not be used for staph or strep species.
Illustration A is a diagram showing the mechanism of action of different antibiotics. Incorrect Answers:
Answer 2: Penicillins work by inhibiting bacterial cell wall synthesis. Answer 3: Rifampin inhibits RNA polymerase
Answer 4: Macrolides bind the 50S-ribosomal subunit Answer 5: Quinolones inhibit DNA gyrase
-
A 25-year-old patient has a blood clotting disorder. In this disorder, the variant co-factor cannot be inactivated, causing overproduction of thrombin. What is this patient's clotting disorder?
-
Protein S deficiency
-
Glanzmann's thrombasthenia
-
Factor V Leiden thrombophilia
-
Antiphospholipid Antibody Syndrome
-
Von Willebrand's disease
CORRECT ANSWER: 3
This patient has Factor V Leiden thrombophilia.
Normal factor V serves as a cofactor to Facotr Xa, and together they produce thrombin and drive coagulation. Protein C helps regulate this process by inhibiting factor V to prevent hypercoaguability. The altered factor V in Factor V Leiden can not be inhibited by protein C, and thus a hypercoaguable state results.
Bertina et al. were the first to report a mutation in blood coagulation factor V and the associated resistance to activated protein C (APC). In addition, they found that 5% of healthy individuals show APC resistance, which is associated with a seven-fold increase in the risk for deep vein thrombosis.
Vosmaer et al. reviewed coagulation abnormalities in Legg-Calvé-Perthes disease. They showed an increased incidence of the factor V Leiden mutations in patients with Legg- Calvé-Perthes disease (odds ratio, 3.3; 95% confidence interval, 1.6 to 6.7).
Illustration A shows the coagulation cascade. Incorrect Answers:
Answer 1: Protein S deficiency acts as a nonenzymatic cofactor to activated protein C. Deficiency results in the inability to activate Protein C, causing overproduction of thrombin.
Answer 2: Glanzmann thrombasthenia is a rare genetic platelet disorder in which the platelets have qualitative or quantitative deficiencies causing increased bleeding.
Answer 4: Antiphospholipid syndrome is an autoimmune disease. These antibodies react against proteins that bind to phospholipids. Hypercoagulation is thought to be related to antibodies against Protein C and possibly Protein S.
Answer 5: Von Willebrand's disease is a genetic bleeding disorder caused by missing or defective von Willebrand factor (VWF), a clotting protein.
-
A 55-year-old male undergoes surgery on his right shoulder. In the preoperative holding area, he reveals that his older brother and mother have had serious problems with general anesthesia in the past. Twenty minutes into the surgery, the surgeon notices skin mottling and increasing limb stiffness, making it hard to manipulate the arm. He calls for muscle relaxation, but is told a depolarizing neuromuscular blocker has already been given. End-tidal CO2 rises significantly, his pulse rate is now 120-130 bpm, and his urine appears dark. In response the anesthesiologist disconnects the vaporizer, begins hyperventilation with oxygen, and stops all triggering agents.
In addition to the steps above, which of the following medications should be administered as an antidote?
-
naloxone
-
dantrolene
-
flumazenil
-
protamine
-
neostigmine
Correct answer: 2
Dantrolene is the antidote for malignant hyperthermia (MH).
Malignant hyperthermia is rare (1:15,000-1/50,000), life threatening condition triggered by volatile anesthetic agents and succinylcholine (a depolarizing neuromuscular blocker). It is an autosomal dominant condition caused by abnormalities in the ryanodine receptor (RYR1) gene. It causes a hypercatabolic state with increased skeletal muscle metabolism, rapid oxygen depletion, increased carbon dioxide concentration and body temperature, leading to circulatory collapse and death.
Glahn et al. described the European Malignant Hyperthermia Group (EMHG)
guidelines for managing malignant hyperthemia.They state that MH is rarer now because of increasing use of total intravenous anesthesia (TIVA) in European countries. The guidelines are seen in Illustrations A and B.
Maclennan et al. describe the mechanism of MH. In MH, the abnormal Ca2+ release channel is more sensitive to stimulators of opening, releases more Ca2+, and does not close readily, flooding the cell with Ca2+ and overpowering the Ca2+ pump. They also found that dantrolene inhibits halothane-induced and Ca2+-induced Ca2+ release from sarcoplasmic reticulum.
Illustrations A and B are the EMHG guidelines for recognizing and managing MH, respectively.
Incorrect Answers:
Answers 1: Naloxone is an opioid antagonist and is used as the antidote for acute opioid overdose and in reducing the respiratory or mental depression due to opioids.
Answer 3: Flumazenil is a GABA receptor antagonist used as an antidote in the treatment of benzodiazepine (BDZ) overdose. It acts by competitive inhibition at the BDZ binding site on the GABA receptor.
Answer 4: Protamine sulfate is used as the antidote for heparin overdose. Answer 5: Neostigmine is used to reverse the effects of non-depolarizing muscle relaxants such as rocuronium and vecuronium at the end of an operation. It acts by preventing breakdown of acetylcholine, thus indirectly stimulating nicotinic and muscarinic receptors.
-
A 52-year-old woman presents with longstanding deformities in both hands. A photograph of her fingernail is shown in Figure A. Which set of radiographs shown in Figures B through F would best
correspond to this clinical appearance?
-
Figure B
-
Figure C
-
Figure D
-
Figure E
-
Figure F
CORRECT ANSWER: 3
This patient has onycholysis and pitting of the nail plate, characteristic of psoriasis. A characteristic hand deformity seen in psoriatic arthritis of the hands is seen in Figure D.
The characteristic psoriatic deformity is arthritis mutilans with opera glass deformity (la main en lorgnette) or telescoping joint destruction, so named because excess skin from the shortening of the phalanx bones becomes folded transversely, as if retracted into one another like opera glasses (retractable telescoping binoculars). Other findings in psoriatic arthritis of the hands
include fluffy periostitis (caused by periosteal ossification), pencil-in-cup deformity (simultaneous destruction of the head of the middle phalanx and expansion of the base of the distal phalanx), acro-osteolysis (resorption of the distal phalanx tuft) and flail digits.
Day et al. reviewed psoriatic arthritis. This disease affects 6-48% of patients with psoriasis. Arthritis mutilans is found in 5% of patients and is destructive in 50%. Following surgery, wound infection rates are higher (skin plaques harbor bacteria) and wound healing may be complicated by the Koebner phenomenon (psoriatic outbreak following trauma) although healing rates are the same as normal skin. Periarticular bone loss may make reconstructive procedures challenging.
Figure A shows nail pitting and onycholysis (incomplete lifting of the nail plate starting at the distal free edge). Illustration A is a table showing the Psoriatic arthritis study group criteria for diagnosing psoriatic arthritis (3 or more points is indicative of psoriatic arthritis).
Incorrect Answers:
Answer 1: Figure B shows rheumatoid hands. There is a predilection for involvement of the PIPJ and MCPJ (especially 2nd and 3rd MCPJ), ulnar styloid and triquetrum, and sparing of the DIPJ. Characteristic findings seen include subchondral cyst, ulnar drift of the MCPJ, boutonniere and swan neck deformities, hitchhiker’s thumb deformity, carpal instability with scapholunate dissociation, and ulnar-carpal ankylosis.
Answer 2: Figure C shows erosive osteoarthritis. The DIPJ, PIPJ and 1st CMCJ are involved. Characteristic findings seen include diffuse cartilage space loss, subchondral erosions (at least 2 central erosions affecting separate IP joints), central erosions producing "gull wing" appearance, absence of marginal erosions, fusiform soft-tissue swelling and osteopenia.
Answer 4: Figure E shows the hands of a patient with gout. Characteristic findings include periarticular osteopaenia, eccentric erosions (well-defined marginal/juxta- articular punched-out rat-bite erosions with sclerotic margins with overhanging edges, and periarticular soft tissue swelling and tophi deposition. Tophi can calcify in the presence of renal disease.
Answer 5: Figure F is a radiograph of a patient with Dupuytren's disease with contractures of the small finger MCPJ and PIPJ.
-
A 23-year-old male grabs at his opponent during a football game. He complains of pain and swelling in his ring finger. Figures A and B are low and high magnification sagittal ultrasound images of the digit, and Figure C is the corresponding sagittal MRI image. An avulsed bone fragment is labelled with a solid white arrow in the ulrasound images, and a long red arrow on the MRI. In this injury pattern, the blood supply to the flexor digitorum profundus (FDP) would be most compromised due to injury to which of the following anatomic structures?
-
Vincula brevia to flexor digitorum profundus (VBP)
-
Vincula longa to flexor digitorum profundus (VLP)
-
Vincula brevia to flexor digitorum superficialis (VBS)
-
Vincula longa to flexor digitorum superficialis (VLS)
-
Volar plate
CORRECT ANSWER: 1
This patient has sustained a Leddy Packer type II jersey finger with retraction of the FDP tendon with a avulsed bone fragment at the level of the distal A2 pulley. In these injuries, the Vincula brevia to flexor digitorum profundus
(VBP) is disrupted, and so is the blood supply to the FDP as the vincula are the main blood supply FDP.
The synovial tendons are nourished by vessels from 3 areas: the musculotendinous junction; the osseotendinous junction; and vessels from various surrounding connective tissue such as the paratenon, mesotenon and vincula. The main blood supply to the bulk of sheathed tendons is from the vincula. The FDS and FDP receive blood from both the vincula brevia and vincula longa. There are 5 vincula - 2 short (brevia) and 3 long (longa).
For the FDS, the vincula brevia superficialis (VBS) arises from the level of the PIPJ and adjacent proximal phalanx. The vincula longa superficialis (VLS) arises from the level of the proximal end of the proximal phalanx and MCPJ, and branches into 2 slips to attach to the tendon. For the FDP, the vincula brevia profundus (VBP) arises from the level of the DIPJ and distal portion of
the middle phalanx and attaches to the FDP at the A5 pulley. The vincula longa profundus (VLP) arises from the distal end of the proximal phalanx and volar plate of the PIPJ and passes through the VBS to attach to the FDP..
Fenwick et al. reviewed the blood supply of tendons. They found that vessels are arranged longitudinally within the tendon, passing around the collagen fibre bundles in the endotenon, a sheet of loose connective tissue contiguous
with the external epitenon. Vessels at the bone insertion do not pass directly through from bone into the tendon due to the interposed fibrocartilage layer, but anastomose with those in the periosteum, forming an indirect link with the osseous circulation.
Figures A and B show type II jersey finger with retraction of the FDP tendon with a large bone fragment avulsion to the level of the distal A2 pulley. Figure C is a sagittal T2- weighted image showing the proximally retracted FDP surrounded by fluid at the level of the proximal phalanx head (arrow). The radial slip of the FDS (short arrows) follows a normal course toward its insertion at the middle phalanx. Illustration A is a schematic diagram depicting the relative positions of the phalanges, FDP and avulsed cortical fragment. Illustration B demonstrates the vincula tendina of the flexor tendons. Illustration C is a table showing the vascular disruptions in jersey finger. Illustration D comprises axial T1-weighted (left) and fat-suppressed proton density-weighted (right) images at the level of the middle phalanx
demonstrate 2 FDS slips (arrows) with no FDP.
Incorrect Answers:
Answers 2: In a Leddy Packer type II jersey finger with retraction of the FDP tendon with a bone fragment avulsion to the level of the distal A2 pulley the Vincula longa to flexor digitorum profundus (VLP) is not damaged and remains intact along with its blood supply to FDP.
Answers 3 and 4: Vincula brevia (VBS) and Vincula longus (VLS) to flexor digitorum superficialis would not be affected in a Jersey finger as the FDS is not involved.
Answer 5: The volar plate is not a a blood supply to the FDP tendon.
-
A 63-year-old female presents with the injury seen in Figure A after a ground level fall. Figure B is her postoperative radiographs. During follow-up she complains of pain that seems to be out of proportion. On physical exam she is noted to have swelling and trophic changes about her skin. Her current WBC count, ESR and CRP are within normal limits. There is no drainage from the pin sites. What postoperative medication protocol has been shown to prevent this complication?
-
Cephalexin 500mg BID for 10 days
-
Allopurinol 200 mg for 14 days
-
Vitamin C 200 mg for 14 days
-
Vitamin C 500 mg for 50 days
-
Indomethacin 75 mg for 10 days
CORRECT ANSWER: 4
This patient is presenting with complex regional pain syndrome (CRPS) following a distal radius fracture treated in an external fixator. Vitamin C 500mg for 50 days has been shown to decrease the risk of developing this complication.
Complex regional pain syndrome is characterized by pain out of proportion, swelling, stiffness, skin discoloration, vasomotor disturbance, trophic skin changes, and hyperhidrosis. CRPS can be difficult to treat, and thus preventative measures are important. Vitamin C has been shown to be effective in decreasing the risk of CRPS following wrist fractures (as well as foot and ankle injuries). The recommended dosing protocol is 500 mg daily for
50 days.
Shibuya et al. present a meta-analysis of the use of vitamin C to prevent CRPS and found a decreased risk of CRPS when vitamin C was started immediately following surgery and continued for 50 days. All papers in the analysis involved injuries to the wrist except for one, which involved injuries to the foot and ankle.
Zollinger et al. present a double-blind prospective study on the use of vitamin C following wrist fractures with different dosing protocols. They found a significant decrease in the risk of developing CRPS with the use of vitamin C, and recommend an effective dose of 500mg for 50 days.
Figure A demonstrates an intraarticular distal radius fracture. Figure B is a postoperative radiograph of the distal radius after placement of an external fixator and kirschner wire.
Incorrect Answers:
Answer 1: Cephalexin is a common antibiotic for infections such as cellulitis. While cellulitis may occur following placement of an external fixator, this patients ESR and CRP and WBC are normal, and it is unlikely to be her diagnosis.
Answer 2: Allopurinol is a treatment for chronic gout.
Answer 3: Vitamin C 200 mg for 14 days would not decrease the risk of CRPS. Answer 5: Indomethacin is an antinflammatory drug used to prevent heterotopic ossification.
-
A resident arrives to the operating room late for a LEFT carpal tunnel release procedure. The patient is prepped and draped under general anaesthesia, and the attending surgeon and assistant are about to make an incision to the RIGHT carpal tunnel. The resident mentions to the attending surgeon that surgery was booked for the opposite limb, but he ignores the residents confers by saying the surgical mark is under the drape. What would be the most appropriate course of action for the resident at this time?
-
Ask the nurse to call the medical director
-
Ask the surgeon to revisit the surgical safety checklist
-
Assume the booking form was incorrect and proceed with the operation
-
Leave the operating room to ask a family member
-
Read the last clinical note for clarification
CORRECT ANSWER: 2
The most appropriate course of action for the resident at this time would be to perform a surgical timeout prior to the operation.
Wrong-site surgery is completely preventable by having the surgeon, in consultation with the patient when possible, place his or her initials on the operative site using a permanent marking pen and then operating through or adjacent to his or her initials. The intended site should be marked such that the mark will be visible after the patient has been prepped and draped. A "time-out" procedure should be done before the initiating of any surgical procedure to confirm the type of procedure, site, and side with all operating room personnel including residents.
Haynes et al. reviewed the effect of surgical safety checklists before surgery on the morbidity and mortality in a global population. They found the rate of death was 1.5%
before the checklist was introduced and declined to 0.8% afterward (P=0.003). Inpatient complications occurred in 11.0% of patients at baseline and in 7.0% after introduction of the checklist (P<0.001).
Incorrect Answers:
Answers 1,3,4,5: The WHO safety checklist requires all personnel to be involved.
Details of the checklist should include, correctly confirming the patients identity, surgical site, equipment, anticipated surgical events, anesthesia check list, and nursing checklist.
-
Which of the following points labeled on Figure A represents the yield point?
-
A
-
B
-
C
-
D
-
E
CORRECT ANSWER: 3
In Figure A, the yield point is represented by point C.
The yield point is defined as the point beyond which permanent deformation
occurs. Prior to the yield point, the material will deform elastically and will return to its original shape when the applied stress is removed. Once the yield point is passed, some fraction of the deformation will be permanent and non- reversible. This is referred to as the "plastic" zone. Prior to this point, the material is in the "elastic" zone. The relative value of the yield point, or yield strength, for various metals is as follows: cast cobalt chrome > titanium > wrought cobalt chrome, stainless steel.
Kennedy et al. determined the ultimate failure strengths of the MCL, ACL, and PCL. The found the PCL had the highest ultimate failure strength, while the ACL and MCL had roughly equal ultimate failure strengths. In addition, they found that microscopic failure is evident prior to gross, macroscopic failure.
Illustration A shows a labeled stress/strain curve. Not the yield point is at the junction of the elastic and plastic zones.
Incorrect Answers:
Answer 1: Point A represents the breaking point, or fracture point. This is the point at which the object fails.
Answer 2: Point B represents the ultimate strength. Ultimate strength is the maximum stress that a material can withstand before failing or breaking. Answer 4: Point D represents Young's modulus of elasticity. It is a measure of the stiffness (ability to resist deformation) of a material in the elastic zone. Answer 5: Point E is the "zero" point. No stress is being applied so there is no
associated strain.
-
What term in statistics defines rejecting the null hypothesis when it is in fact true?
-
Type-I error
-
Type-II error
-
Confounding error
-
Variance
-
Negative likelihood ratio
CORRECT ANSWER: 1
Rejecting the null hypothesis when it is true is an example of a type-I error.
Type-I errors, to put it simply, detect an effect that is not present. In contrast, a type-II error fails to detect an effect that is present. In simple studies, the rate of a type-I error is denoted by a (alpha). For a 95% confidence level, the value of alpha is 0.05. This means that there is a 5% probability that we will reject a true null hypothesis.
Illustration A shows the difference between type-I and type-II errors. Incorrect Answers;
Answer 2: A type-II error accepts the null hypothesis when it should be rejected.
Answer 3: A confounder is a variable that has associations with both the dependent and independent variables, potentially distorting their relationship. Confounders are not technically considered "errors," but instead are variables that properly constructed studies attempt to avoid.
Answer 4: Variance is an estimate of the variability of each individual data point from the mean.
Answer 5: Negative likelihood ratio describes how the likelihood of a disease is changed by a negative test result.
-
A funnel plot is used in meta-analyses to perform which of the following functions:
-
Illustrate the relative strength of treatment effects in multiple studies
-
Detect publication bias
-
Graph of the sensitivity versus 1-specificity of a diagnostic test
-
Determine the sample size required to detect an effect of a given size with a given degree of confidence
-
Predict the unknown value of a variable from the known value of two or more variables
CORRECT ANSWER: 2
A funnel plot is the most commonly used statistical test for detection of publication bias in meta-analyses.
Publication bias occurs because studies with a non-significant result, so-called negative studies, have a higher likelihood of being rejected than positive studies, and are oftentimes not even submitted for publication. Funnel plots, which plot the effect size of a study against a measure of the study’s size are used to detect this bias. This method is based on the fact that larger studies have smaller variability, whereas small studies, which are more numerous, have larger variability. Thus the plot of a sample of studies without publication bias will produce a symmetrical, inverted-funnel shaped scatter, whereas a biased sample will result in a skewed plot.
Vavken et al. reviewed orthopaedic meta-analyses in order to determine whether publication bias was assessed and to evaluate its effect on the outcomes of these meta- analyses. They found that only 35% of all orthopaedic meta-analyses published between 1992 and 2008 in English and German assessed publication bias. Adjustment for publication bias did not produce significantly different results, but the magnitude of the pooled estimates in the affected meta-analyses changed by 29% on average.
Illustration A depicts a symmetrical funnel plot with no evidence for publication bias.
Illustration B shows a skewed funnel plot suggesting publication bias, as
it is missing studies in the lower left corner, i.e. ‘‘negative studies’’. Illustration C depicts a forest plot comparing the incidence of squeaking between ceramic- on-ceramic (COC) and ceramic-on-polyethylene (COP). Illustration D is an example of a ROC curve examining the probability of DVT.
Incorrect Answers:
Answer 1: Forest plots illustrate the relative strength of treatment effects in multiple studies, NOT funnel plots.
Answer 3: A graph of the sensitivity versus 1-specificity of a diagnostic test is referred to as a receiver operating characteristic (ROC) curve and is typically used to determine the accuracy of diagnostic tests.
Answer 4: A power analysis is used to determine the sample size required to detect an effect of a given size with a given degree of confidence prior to the start of a clinical study.
Answer 5: A multiple regression analysis is used to predict the unknown value of a variable from the known value of two or more variables in a study.
-
Which of the following Figures represents a disease process that is caused by over-secretion of hormone that preferentially affect the proliferative zone of the growth plate?
-
Figure A
-
Figure B
-
Figure C
-
Figure D
-
Figure E
CORRECT ANSWER: 2
Figure B shows a picture of a male with Gigantism (over-secretion of growth hormone) who had a final height of nearly 9 feet. Longitudinal bone growth
occurs at the growth plate by a process called endochondral ossification in which cartilage is first formed and then remodeled into bone tissue. The growth plate consists of three principal layers: the resting zone, proliferative zone, and hypertrophic zone.
Growth hormone acts preferentially at the proliferative zone where it stimulates longitudinal bone growth.
Figure A represents Spondyloepiphyseal dysplasia (SED) where there is disproportionate dwarfism, spinal involvement, and a barrel chest from a COL2A1 mutation. Figure C represents diastrophic dysplasia with a "hitch- hikers" thumb, "cauliflower ear", cleft palate, and short-limbed dwarfism due
to a sulfate transport mutation. Figure D represents cleidocranial dysplasia due to defect in core-binding factor alpha 1 (CBFA-1) causing dwarfism and absent clavicles. Figure E represents Multiple epiphyseal dysplasia (MED) causing disproportionate dwarfisim with multiple epiphyses involved, shortened metacarpals, valgus knees, but no spinal involvement, all of which are due to
a COMP mutation.
-
Which of the following is a function of siRNA (small interfering RNA)?
-
Identifies proteins
-
Blocks transcription of DNA
-
Separates DNA based on size
-
Blocks translation of mRNA
-
Identifies DNA sequences
CORRECT ANSWER: 4
siRNA functions by causing mRNA to be broken down after transcriptions, resulting in an inability to undergo translation.
siRNA are short (usually 20 to 24-bp) double-stranded RNA (dsRNA) sequences with phosphorylated 5' ends and hydroxylated 3' ends. Because of their ability to block a gene of interest, they have been generating interest in the treatment of disease processes that involve gene expression.
Noh et al. study the affects of PD98059, an extracellular signal-regulated kinase 1/2 (ERK1/2) inhibitor, on osteosarcoma. They found that blocking the ERK1/2 pathway with PD98059 induces osteosarcoma cell death by inhibiting a potential drug-resistance mechanism.
Illustration A shows how siRNA works to block translation of mRNA. Incorrect Answers:
Answer 1: This is the function of a Western blot.
Answer 2: siRNA have not been shown to block transcription of DNA. Answer 3: This is the function of a gel electrophoresis.
Answer 5: This is the function of a Southern blot.
-
Regarding bone erosion in rheumatoid arthritis, which of the following statements is true?
-
Interference with Wnt signalling may reduce bone erosion
-
TNF and IL-6 blockade leads to slowing of bone erosion
-
The extent of bone erosion is independent of the extent of synovitis
-
M-CSF and RANKL stimulate bone resorption by synovial fibroblasts
-
The presence of serum anti-citrullinated protein antibodies is predictive of the extent of synovitis but not bone erosion
CORRECT ANSWER: 2
TNF, IL1 and IL-6 receptor blockade helps to slow/arrest bone erosion in RA and is also effective in reducing synovitis.
Cytokines TNF, IL-1 and IL-6 are key players in RA. TNF stimulates migration of osteoclast precursors from the bone marrow into the periphery, and stimulates expression of surface receptors to facilitate differentiation. In the joint, M-CSF and RANKL stimulate differentiation towards osteoclasts. Final differentiation into bone- resorbing osteoclasts is achieved following contact with the bone surface.
Schett et al. reviewed bone erosions in RA. They state that the main triggers
of bone erosion are synovitis, RANKL, and anti-citrullinated protein antibodies. In RA, there is an abundance of osteoclasts in bone erosions, but a paucity of mature osteoblasts, suggesting the presence of molecules that block osteoblast differentiation.
Ideguchi et al. investigated whether repair of erosions occurs in patients with rheumatoid arthritis (RA) treated with conventional disease-modifying anti- rheumatic drugs (DMARDs). They detected repair of erosions in 10.7% of RA patients treated with DMARDs. They recommend the use of DMARDs to reduce disease activity and thus reduce erosions.
Illustration A shows the action of antirheumatic drugs on osteoclast differentiation and bone erosion.
Answer 1: Interference with Wnt signalling impairs bone formation, thus INCREASING bone erosion. The Wnt signaling pathway is a bone anabolic pathway. TNF induces expression of Dickkopf-related protein 1( Dkk-1) in synovial fibroblasts. Dkk-1 interferes with Wnt signalling and blocks new bone formation. Other Wnt antagonists Frizzled- related protein-1 and sclerostin also inhibit bone formation and inhibit repair of bone erosion.
Answer 3: The greater the synovitis, the more extensive the bone erosive process. Some anti-rheumatic drugs (steroids, methotrexate, leflunomide) are bone sparing simply because they reduce synovitis.
Answer 4: M-CSF and RANKL stimulate osteoclast differentiation from monocyte-lineage cells. Blockage of RANKL by denosumab slows bone erosion (but does not retard inflammation).
Answer 5: Anti-citrullinated protein antibodies (ACPA) are produced by plasma cells and can appear long before the onset of synovitis, and independently predicts bone erosion. ACPA bind to citrullinated vimentin in osteoclasts, stimulating TNF release, and enhancing differentiation into mature osteoclasts.
-
A patient undergoing joint arthroplasty is put on a drug that competitively inhibits the activation of an enzyme that breaks down Factor Ia. The drug is
-
Dabigatran
-
Tranexamic acid
-
Rivaroxaban
-
Fondaparinux
-
Heparin
CORRECT ANSWER: 2
Factor Ia is fibrin. The enzyme that breaks down fibrin is plasmin. Tranexamic acid (TXA) is an antifibrinolytic that prevents the activation of plasmin from the inactive zymogen plasminogen.
Tranexamic acid competitively inhibits the activation of plasminogen to plasmin by binding to specific sites on both plasminogen and plasmin. Tranexamic acid has roughly eight times the antifibrinolytic activity of an older analogue, e-aminocaproic acid. It is used during joint replacement surgery to reduce blood loss and the need for transfusion.
Watts et al. review strategies for minimizing blood loss and transfusion. They recommend 1g of TXA prior to incision, and 1g at wound closure. They also recommend giving fluids for symptoms of anemia, rather than transfusion, as even high risk patients do well with sufficient intravascular volume even with low hemoglobin levels.
Imai et al. evaluated TXA in 107 patients undergoing THA. They found that intraoperative blood loss after preoperative TXA administration was lower than both control and postoperative TXA administration groups. They recommend using 1 g of TXA 10 minutes before surgery and 6 hours after the first administration to best reduce blood loss during THA.
Gillette et al. retrospectively reviewed 2046 patients receiving TXA for THA or TKA together with either aspirin, warfarin or dalteparin. They found that the rates of symptomatic DVT (0.35%, 0.15%, and 0.52%, respectively) and nonfatal PE were similar (0.17%, 0.43%, and 0.26%, respectively) for the 3 drugs respectively. They recommend TXA to decrease blood loss and transfusion.
Illustration A shows the role of tranexamic acid in the fibrinolytic cycle and the clotting cascade.
Incorrect Answers:
Answer 1: Dabigatran is an oral direct thrombin inhibitor.
Answer 3: Rivaroxaban is an oral direct factor Xa inhibitor. It is FDA approved for hip and joint arthroplasty DVT prophylaxis.
Answer 4: Fondaparinux is an indirect factor Xa inhibitor that works through antithrombin
III. Unlike heparin, it is selective for factor X and does not have effect on thrombin. Answer 5: Heparin is an anticoagulant that binds and activates antithrombin
III. Activated antithrombin III then inactivates thrombin and factor Xa.
-
Figure A demonstrates the molecular structure of a cell membrane protein
important in propagating the action potential of a neuron. Which of the following medications acts by binding to the
location marked with an X in the illustration?
-
Phentolamine
-
Rocuronium
-
Bupivacaine
-
Midazolam
-
Fentanyl
CORRECT ANSWER: 3
Bupivacaine exerts its actions through blockade of the voltage gated sodium channel.
Local anesthetics of the amide family (lidocaine, bupivacaine) bind to the intracellular portion of voltage-gated sodium channels to block sodium influx. This prevents depolarization and the initiation or conduction pain. Lidocaine and bupivacaine have a duration of action of 2 and 4 hours, respectively (4 and 8 hours with epinephrine), and maximum doses are 4.5mg/kg and
2.5mg/kg, respectively (7mg/kg and 3mg/kg with epinephrine respectively). Both are metabolized in the liver, and excreted by the kidneys.
Phillips et al. review specific analgesics. Agents used to manage chronic pain include tricyclic antidepressants, anticonvulsants, GABA agonists, local anesthetic analogs, and NMDA antagonists. Opiates may trigger tolerance and lack of efficacy may develop. In those with refractory chronic pain, centrally administered analgesics may be considered, including opiates, dilute local anesthetic, NMDA receptor antagonists, clonidine, midazolam, baclofen, or calcium channel blockers. Single agents may be less effective than analgesic combinations.
Scholz discussed the function of local anesthetics and sodium channels. There are 3 states to sodium channels: (1) The closed state at potentials below
-70mV. In this state, Na+ ions cannot pass from 1 side to another. (2) The open state, initiated by depolarization of the membrane to above -40mV. The channel opens to allow
Na+ ions to diffuse through the pore, causing an inward current, depolarizing the membrane further. (3) The inactivated state follows activation during prolonged depolarization. In this state, inactivation is seen in macroscopic currents.
Figure A shows the voltage gated sodium channel. Local anesthetics such as lidocaine and bupivacaine act at the binding site marked "X".
Incorrect Answers:
Answer 1: Phentolamine competitively blocks alpha-adrenergic receptors and leads to vasodilatation. It antagonizes circulating epinephrine and/or norepinephrine.
Answer 2: Rocuronium is a non-depolarizing neuromuscular blocking agent. It acts by competitively blocking the binding of acetylcholine to its receptors.
Answer 4: Midazolam increases the efficiency of GABA to decrease the excitability of neurons. GABA receptors contain a chloride ion channel. By binding to the GABA receptor, midazolam increases the influx of chloride ions, hyperpolarizing the neuronal cell membrane, decreasing its excitability. Answer 5: Fentanyl exerts its activity through binding to µ and kappa receptors. The supraspinal analgesic properties are mediated by the µ1 receptor, respiratory depression and physical dependence by the µ2 receptor, and sedation and spinal analgesia by the kappa receptor.
-
Which of the following defines the incidence of a disease?
-
The total number of cases of a disease in a city
-
The number of new cases of a disease diagnosed during a specific time period
-
The average number of cases of a disease per year over the last 10 years
-
The number of existing cases of a disease divided by total population in a city
-
The variability in the total number of disease cases between major US cities
CORRECT ANSWER: 2
As described in the review by Kocher and Zurakowski, incidence is defined as the number of NEW cases of a disease diagnosed during a specific time period per unit measurement of population. This is different from the prevalence of a disease, which is defined as the total number of cases of the disease in the population at a given time. The variance of a disease is an estimate of the variability of each individual data point from the mean, so how each patient with a disease differs from the mean of patients with that disease (e.g. length of symptoms of patients with a disease may differ for individual patients, but a mean duration of symptoms exists for all patients with that disease).
Kocher and Zurakowski present a Level 5 review article that is referenced often and discusses basic biostatistic principles.
-
Which of the following cells are NOT components of the innate immune system?
-
Dendritic cells
-
Natural killer cells
-
Basophils
-
CD8+ T lymphocytes
-
Neutrophils
CORRECT ANSWER: 4
CD8+ T lymphocytes are not part of the innate immune system.
Innate immunity refers to nonspecific defense mechanisms that come into play immediately or within hours of an antigen's appearance in the body. The cells of this system include natural killer cells, mast cells, eosinophils, basophils and phagocytic cells (macrophages, neutrophils, and dendritic cells). Metal-on- metal bearing reactivity is related to the innate immune response (cell mediated). Adaptive immunity refers to antigen-specific immune response.
The adaptive immune response is more complex as the antigen must first be processed and recognized. After antigen recognition, the adaptive immune system creates an army of immune cells specifically designed to attack that antigen. Adaptive immunity includes "memory" that makes future responses against the same antigen more efficient. The cells of this system include CD8+ T lymphocytes, T helper cells, delta gamma T cells, B cells and plasma cells.
Landgraeber et al. review the pathology of orthopaedic implant failure. They found that aseptic loosening is primarily mediated by the innate immune system. Macrophages respond to wear debris with both damage (DAMP, damage associated molecular pattern) and pathogen (PAMP, pathogen associated molecular pattern) cytokine signalling. Macrophage cytokines
include IL-1a, IL-1ß, IL-6, IL-10, IL-11, IL-15, TNF-a, TGF-a, GM-CSF, M-CSF), PDGF and EGF.
Illustration A shows the innate immune system. Incorrect Answers:
Answers 1, 2, 3, 5: These cells belong to the innate immune system.
-
What is the main biologic effect of aggrecan in cartilage?
-
Extracellular matrix protein involved in the organization of collagen
-
Proteoglycan involved in the hydrophilic behaviour of cartilage
-
Cartilage matrix protein that plays a role in cartilage tissue organization
-
Collagen component responsible for stability
-
Non-collagenous extracellular matrix protein that regulates chondrocyte proliferation
CORRECT ANSWER: 2
Aggrecan binds hyaluronic acid to attract water, which accounts for its hydrophilic property.
Aggrecan is the predominant proteoglycan in cartilage. It contains a large number of negatively charged sequences that attract water called sulfated glycosaminoglycan (GAG) chains. Its the N-terminal globular domain of aggrecan that binds hyaluronan to form huge aggregates. Together with its chondroitin sulfate chains, they help to create a hydrophilic viscous gel that decreases the coefficient of friction as well as to help absorb compressive loads.
Ulrich-Vinthe et al. reviewed the biology of articular cartilage. They report that
matrix metalloproteinases and aggrecanases play a major role in aggrecan degradation and their production is upregulated by mediators associated with joint inflammation and overloading.
Illustration A shows a depiction of the function of aggrecan in articular cartilage. In the relaxed state, the aggregates draw water into cartilage. With compressive loads, the water is displaced to cushion the load. Upon removal of the load, the water content is restored.
Incorrect Answers:
Answer 1: This describes the biological effect of Decorin. Answer 3: This describes the biological effect of Matrilin 1. Answer 4: This describes the biological effect of Type IX collagen
Answer 5: This describes the biological effect of Cartilage oligomeric matrix protein.
-
Which of the following places materials in order of increasing Young's modulus of elasticity (from least stiff to most stiff)?
-
Titanium < Cortical bone < Stainless Steel < Cobalt-chromium alloy
-
Cortical bone < Titanium < Stainless Steel < Cobalt-chromium alloy
-
Titanium < Stainless Steel < Ceramic < Cobalt-chromium alloy
-
Stainless Steel < Titanium < Ceramic < Cobalt-chromium alloy
-
Stainless Steel < Titanium < Cobalt-chromium alloy < Ceramic
CORRECT ANSWER: 2
An increasing Young's modulus of elasticity is seen in order going from cortical bone, to titanium, to stainless steel, to cobalt-chromium alloy, to ceramic (AL2O3).
Material properties are an important consideration in orthopaedic implants. One such property is the Young's modulus of elasticity, which is a
measurement designed to quantify the stiffness of materials. The formula used to calculate Young's modulus is, "stress" (force applied) divided by "strain" (degree of displacement).
Thus, a material may have a higher Young's
module, and be considered "more stiff", when a greater amount of stress is needed to produce a similar amount of strain in other materials that are considered "less stiff".
Mann et al. looked at the properties of common orthopaedic implants. Titanium is notable for having the most similar Young's modulus compared to cortical bone. Stainless steel is more stiff than titanium and also commonly used in orthopaedic implants. Cobalt- chromium alloys are known for being the most stiff among metals found in orthopedic implants, and are known for
being highly resistant to corrosion. Ceramic is even more stiff than cobalt- chromium alloy.
Illustration A represents the formula for calculating Young's modulus of elasticity. Illustration B is a list of materials encountered in orthopaedic surgery in order of their Young's modulus.
Incorrect Answers:
Answer 1: Titanium has a higher Young's modulus than cortical bone
Answer 3: Ceramic has a higher Young's modulus than cobalt-chromium alloy Answer 4: Stainless steel has a higher Young's modulus than titanium Answer 5: Stainless steel has a higher Young's modulus than titanium
-
Type I collagen fibers in peripheral nerves are primarily responsible for which of the following?
-
Conduction strength
-
Tensile strength
-
Conduction velocity
-
Compressive strength
-
Cross-linkage
CORRECT ANSWER: 2
Type I collagen fibers are most responsible for the tensile strength of a peripheral nerve.
Type I collagen is the most abundant collagen of the human body which forms large, eosinophilic fibers known as collagen fibers. It is present in scar tissue, the end product when tissue heals by repair, as well as tendons, ligaments, the endomysium of myofibrils, the organic part of bone, the dermis, the dentin and organ capsules.
The COL1A1 gene produces a component of type I collagen, called the pro- alpha1(I) chain. This chain combines with another pro-alpha1(I) chain and also with a pro- alpha2(I) chain (produced by the COL1A2 gene) to make a molecule of type I procollagen. These triple-stranded, rope-like procollagen molecules must be processed by enzymes outside the cell. Once these molecules are processed, they arrange themselves into long, thin fibrils that cross-link to one another in the spaces around cells. The cross-links result in the formation of very strong mature type I collagen fibers.
Wong et al. provide a review of the basic science behind nerve healing and the recovery after nerve repair. They note the importance of minimizing additional surgical insult and careful handling of nerve tissue during repair to optimize outcomes.
Pertici et al. noted that autologous nerve implantation to bridge a long nerve gap presents the greatest regenerative performance in spite of substantial drawbacks. They were able to show improved nerve guided regrowth with a type I collagen matrix conduit as compared to a conduit made of a mix of type I and type III collagen.
Illustration A shows a diagram of type I collagen, showing the rope-like characteristics behind the tensile strength.
Incorrect Answers:
Answer 1: Strength of nerve conduction is not dependant on collagen, but is polyfactorial, with contributions from patient age, sex, nerve temperature, and myelin sheath condition.
Answer 3: Type I collagen does not significantly affect the velocity of nerve conduction, as this is also polyfactorial, as explained in Answer I immediately above.
Answer 4: Type I collagen can be compressed, but is more responsible for the tensile strength through the cross-linkages.
Answer 5: Peripheral nerves do not have cross-linkages.
-
According to patients and colleagues surveyed, orthopaedic surgeons are given the highest ratings for their skills in which of the following areas?
-
Medical knowledge
-
Social gatherings
-
Technical skills
-
Communication
-
Empathy
CORRECT ANSWER: 3
Orthopaedic surgeons are given high ratings by patients and colleagues for their technical skills in the operating room. Patients and colleagues however think orthopaedic surgeons listening and communication skills could be improved upon and they could show more empathy for their patients. Communication affects patient satisfaction, adherence to treatment, physician satisfaction, and is the most common factor in the initiation of malpractice suits.
Tongue et al. review these specific communication data as well as other common communication issues seen in the field of orthopaedic surgery.
-
Your colleague, a general surgeon in your practice, brings his mother-in-law to see you for hip pain. You diagnose her with end- stage hip osteoarthritis and wish to discuss surgical options. She does not speak English. Interpretation should be provided by
-
certified translator
-
patient's husband
-
your colleague
-
patient care advocate fluent in the same language as the patient
-
nurse fluent in the same language as the patient
CORRECT ANSWER: 1
Family members should not be involved with translation for reasons of confidentiality. This is especially true if the patient specifically requests professional translation services.
For patients who have limited English proficiency, translation can be performed by employees fluent in English and a second language, commercial telephonic services, professional translators, or volunteer translators. It is inappropriate for family members to act as translator for reasons of confidentiality, unless
the patient offers or agrees. For medico-legal reasons, certified translators (telephonic or on-site) are preferred.
Tongue et al. reviewed communication skills. They state that the Office for Civil Rights requires physicians who received reimbursement from Medicaid and Medicare Part A to have competent translation services for all patients who have limited English proficiency, and this constitutes "federal financial assistance".
Kaz et al. defined 3 communication barriers between orthopedists and patients: (1) Everyday communication skills must be acquired to effectively interact with patients, (2) There is a growing language divide between patients and physicians who do not share a common language, and (3) Cultural competency issues lead to distrust that can adversely affect patient outcomes.
Incorrect Answers
Answers 2-5: For medico-legal reasons, certified translators (telephonic or on- site) are preferred to other sources.
-
Which of the following fractures (Figures A through F) is representative of a fracture that may be related to long term biphosphonate use?
-
Figure A
-
Figure B
-
Figure C
-
Figure D
-
Figure E
CORRECT ANSWER: 5
Figure E is a bisphosphonate-related subtrochanteric fracture.
Bisphosphonate-related subtrochanteric and femoral shaft fractures are characterized by (1) focal lateral cortical thickening, (2) transverse fracture orientation, (3) medial spike ("beaking") and (4) simple pattern with lack of comminution.
Lenart et al. performed a retrospective case-control study on 41 subtrochanteric fractures. They found that more patients in the subtrochanteric/femoral shaft fracture group were likely to be on bisphosponates than the intertrochanteric/femoral neck group. They identified a common x-ray pattern in 10/15 subtrochanteric fractures on bisphosphonates and called it "simple with thick cortices."
Schilcher et al. reviewed the radiographs of 5342 women and found 172 atypical fractures. The risk was higher with alendronate (relative risk 1.9) than risedronate. Risk increased with duration of use (RR 126 at 4 years).
Compared with men, women had 3.6x greater risk (RR 3.6). They conclude that oral bisphosphonates might do more harm than good if given to patients without indication, and the evidence for prolonged treatment is weak.
Incorrect Answers:
Answers 1 and 2: Figures A and B are non-bisphosponate related subtrochanteric fractures. There is no cortical thickening or fracture site beaking, and these are spiral fractures with more comminution.
Answer 3: Figure C is a comminuted subtrochanteric fracture. The proximal fragment is abducted and the lesser trochanter is fractured and separated from the femur.
Answer 4: Figure D shows a pathological subtrochanteric fracture that is posteriorly displaced and mildly impacted, with lateral angulation of the proximal fragment. There is a lytic lesion at the fracture site from a myelomatous deposit.
-
A 6-year-old girl presents with a lower extremity deformity. Her standing AP radiograph is shown in Figure 1. Her parents report that she has been prescribed vitamin supplements by her prior physician with no improvement in her condition. Calcium, PTH and Vitamin D lab values return within normal limits. What is the most likely inheritance pattern of her disease?
-
Autosomal dominant
-
Autosomal recessive
-
X-linked dominant
-
X-linked recessive
-
Mitochondral
CORRECT ANSWER: 3
The patient most likely presents with X-linked hypophosphatemic rickets, which is inherited in an X-linked dominant pattern.
This inherited disorder is also known as Vitamin D resistant rickets (VDRR), of which the X-linked dominant form is most common. It involves a defect in the absorption of phosphorus by the renal tubules. It is characterized by short stature, genu varum, windswept deformity, physeal widening, low serum phosphorus, elevated alkaline phosphatase, and normal or low normal serum calcium. Other forms of VDRR may be associated with conversion disorders of
25-hydroxyvitamin D to 1,25-hydroxyvitamin D (Type II Vitamin D dependent rickets), in which patients have high 1,25-OH vitamin D, low calcium, and high PTH.
Sharkey et al present a review paper of the management of X-linked hypophosphatemic rickets. They review the medical management as a mainstay which focuses on oral phosphate and vitamin D supplementation, as well as the surgical management which remains somewhat unstandardized, ranging from guided growth to osteotomy with immediate fixation, nailing, or gradual correction with external fixators. Ultimately, the patient's deformity and symptoms guide the specific surgical plan, with most patients being treated medically.
Popkov et al review a series of 47 cases treated with bilateral deformity correction of
femur and tibia with ring fixators and compare this approach to a prior cohort of the same surgeon with staged correction. They found that simultaneous correction reduced the overall duration of treatment with fewer complications and saw no recurrence of any corrections that were ultimately stabilized with intramedullary nailing.
Figure A is a standing AP radiograph of a child with genu varum, physeal widening, and an overall ricketic picture on imaging.
Incorrect Answers:
Answer 1: While a form of VDRR can be inherited as autosomal dominant, this is much rarer than the X-linked dominant form.
Answer 2, 4, 5: The most common form of inheritance of this type of hypophosphatemic rickets is X-linked dominant.
-
Which of the following contributes to the differentiation of mesenchymal progenitor cells towards adipose tissue?
-
Bone morphogenic protein (BMP)
-
Peroxisome proliferator-activated receptor gamma (PPAR-gamma)
-
Runt-related transcription factor 2 (RUNX2)
-
Sex determining region-Y-box 9 (SOX9)
-
Wnt-beta-catenin
CORRECT ANSWER: 2
PPAR-gamma is a regulator of mesenchymal progenitor cell that induces apidocyte formation.
Mesenchymal progenitor cells, often referred to as mesenchymal stem cells (MSCs), are upstream cells that can form myoblasts, chondrocytes, adipocytes, and osteoblasts amongst other tissue types. Numerous regulatory pathways have been explored to understand and manipulate these differentiation pathways. PPAR-gamma has been known for many years to influence adipogenic differentation, most likely at the pre- adipocyte to adipocyte phase. It is inhibited by the Wnt5a pathway which encourages osteoblastic differentiation.
Takada, Suzawa, Matsumoto et al present a basic science paper in which they demonstrate osteoblastic differentiation instead of adipogenic by two different forms of PPAR-gamma repression. The first was through TNF-a and IL-1, with activated NF-kappa beta blocking PPAR-gamma binding to DNA. The second was Wnt-5a repression of PPAR-gamma. Both demonstrated a change from the adipogenic induction typical of PPAR-gamma to osteogenic.
Illustration A from Takada, Kouzmenko, and Kato displays known regulators of MSC differentiation.
Incorrect Answers:
Answer 1 - BMP induces OSTEOGENIC differentiation Answer 3
- RUNX2 induces OSTEOGENIC differentiation Answer 4 - SOX9
induces CHONDROGENIC differentiation
Answer 5 - Wnt-beta-catenin induces OSTEOGENIC differentiation
-
What is the most likely mechanism for failure of a modular revision total hip arthroplasty femoral implant at the mid-stem junction?
-
Electrochemical destruction due to the association of dissimilar metals
-
Fatigue cracks due to differences in oxygen tension
-
Micromotion at the contact site between two components
-
Pit formation causing a decrease in thickness at affected sites
-
Selective attack at the grain boundaries within the metal
CORRECT ANSWER: 3
Fretting corrosion, which describes a mode of destruction from the relative micromotion of two materials, is the most common cause for mid-stem implant failure after total hip arthroplasty (THA) using a modular, revision- type stem.
Fretting is a type of corrosion and wear caused by load and repeated relative micromotion of two implants or implant components. Though an uncommon complication, arthroplasty involving modular implants are at risk for such motion between the components of the final implant because of the increased number of interfaces between the various components. Any of these interfaces can be the site of increased wear and implant failure.
Lakstein et al. analyzed six mid-stem modular implant failures, comparing them to 165 controls. They found evidence of fretting fatigue as well as a bending moment. They also note elevated BMI and inadequate proximal bone stock as risk factors for such failure.
Buttaro et al. present a case of fatigue fracture in a proximally modular, distally tapered fluted implant with diaphyseal fixation. They concluded that fatigue failure was the cause of this complication, emphasizing the reduced proximal femoral bone stock in this patient.
Illustrations A and B, from Lakstein et al, show an AP radiograph and clinical image, respectively, of a modular revision THA with fracture at the mid-stem junction.
Incorrect Answers:
Answer 1 - This describes Galvanic corrosion Answer 2 - This describes Crevice corrosion Answer 4 - This describes Pitting corrosion Answer 5 - This describes Intergranular corrosion
-
Which of the follow medications activates antithrombin III?
-
Warfarin
-
Aspirin
-
Rivaroxaban
-
Dabigatran
-
Heparin
CORRECT ANSWER: 5
Activation of antithrombin (AT) III is the mechanism of action of heparin.
Heparin works by binding to and enhancing the ability of antithrombin III to inhibit factors IIa, III, Xa. It is metabolised by the liver. The risks associated with its use include bleeding and heparin induced thrombocytopenia (HIT). The reversal agent is protamine sulfate.
Brown et al. performed a pooled analysis of 14 randomized controlled trials (RCTs) on VTE rates. They showed that VTE rates with aspirin were not significantly different than the rates for vitamin K antagonists (VKA), low molecular weight heparins (LMWH), and pentasaccharides. They concluded that aspirin may be used for VTE prophylaxis after major orthopaedic surgery.
Murphy et al. reviewed VTE prophylaxis in pediatric patients. Using public health data over 10 years, 285 611 clinical encounters were reported as lower extremity orthopaedic trauma. The incidence of VTE events was 0.058 %. Adolescents and polytrauma patients with injuries of the femur/femoral neck, tibia/ankle, and pelvis are more commonly affected.
Roehrig et al. first examined the coagulation enzyme Factor Xa (FXa) as a target for antithrombotic therapy. This study contributed to the development of rivaroxaban, which is commonly used as an oral antithrombotic agent.
Illustration A shows the interaction between anticoagulant drugs and the coagulation cascade.
Incorrect Answers:
Answer 1: Warfarin inhibits vitamin K 2,3-epoxide reductase, thereby limiting the production of vitamin K-dependent clotting factors (II, VII, IX, X) as well as Protein C and Protein S.
Answer 2: Aspirin inhibits the production of prostaglandins and thromboxanes through irreversible inhibition of cyclooxygenase (COX, 1 and 2) and thus inhibits platelet aggregation.
Answer 3: Rivaroxaban is a direct inhibitor of factor Xa. Answer 4: Dabigatran is a direct thrombin inhibitor.
-
Decorin is a major controlling molecule for:
-
Myelin diameter
-
Non-mineralized bone matrix density
-
Tendon collagen fiber size
-
Elastic cartilage thickness
-
Axon branching pattern
CORRECT ANSWER: 3
Decorin is a proteoglycan molecule that regulates the assembly of collagen fibrils in tendons.
Tendons are composed organized bundles of uniaxially arranged collagen fibrils, which are assembled together to generate force from muscles to bone. However, the variability in tendon morphology is multifactorial. At the microscopic level, decorin is an important regulator of tendon structure as it acts as a mediator of fibril growth. Variations in expression of decorin has shown to correlate with both size and density of collagen fibrils and organisation.
Robinson et al. measured the mechanical properties of multiple tendon tissues in mice with knock-out models of decorin. They found that the loss of decorin affected the posterior tibialis causing an increase in modulus and stress relaxation, but had little effect on the flexor digitorium longus. They concluded
that tendons likely are uniquely tailored to their specific location and function at the microscopic level.
Zhang et al. performed a biomechanical study on tendon function. They showed that decorin-deficient mice demonstrate altered fibril structure and significantly reduced strength and stiffness. They concluded that decorin is a key regulatory molecule in tendon development.
Incorrect Answer:
Answer 1: Myelin diameter is organized by Schwann cells
Answer 2: Non-mineralized bone matrix is formed by osteoblasts. This includes alkaline phosphatase, osteonectin, osteocalcin, etc.
Answer 3: Elastic cartilage is formed from elastic microfibril, which consist of numerous proteins such as microfibrillar-associated glycoproteins, fibrillin, fibullin, and elastin.
Answer 5: Laminin, fibronectin, tenascin, and perlecan are molecules shown to be associated with axonal growth and assembly. Decorin in not a major controlling molecule in the branching pattern.
-
Which of the following medications inhibits release of neurotransmitters by binding to presynaptic calcium channels?
-
Denosumab
-
Sertraline
-
Tramadol
-
Gabapentin
-
Linezolid
CORRECT ANSWER: 4
Gabapentin acts by inhibiting presynaptic calcium channels, thus preventing the release of neurotransmitters.
Gabapentin (also known as Neurontin) is a medication that is commonly used to treat neuropathic pain. It acts by binding the alpha2delta subunit of
voltage-dependent calcium channels on the presynaptic membrane. This serves to increase GABA synthesis, as well as inhibit the release of excitatory neurotransmitters. These neurotransmitters are believed to be part of the pathway leading to neuropathic pain.
Bennett et al. provide a review of the pharmacology of gabapentin for the use of neuropathic pain. They note effective antihyperalgesic and antiallodynic
properties of gabapentin but not significant anti-nociceptive action. Among patients with neuropathic pain they found an average pain score reduction of
2.05 points on an 11 point Likert scale, which compared favorably to placebo.
Mehta et al. explored outcomes of gabapentin and pregabalin (Lyrica) for use in patients with spinal cord injury. Both agents were found to decrease pain and secondary conditions such as sleep disturbance. They did not directly compare these agents to other analgesic medications.
Guy et al. present a meta-analysis of the use of anticonvulsants (such as gabapentin) to treat pain in patients with spinal cord injury. Large effect size was seen in 4 of 6 studies looking at the effectiveness of gabapentin.
Illustration A show the mechanism of currently available antiepileptic drugs (AEDs) that target several molecules at the excitatory synapse. Gabapentin and pregabalin bind to the a2d subunit of voltage-gated Ca2+ channels, which is thought to be associated with a decrease in neurotransmitter release.
Incorrect Answers
Answer 1: Denosumab works by inhibiting the receptor activator of nuclear factor kappa beta ligand (RANK-L)
Answer 2: Sertraline is a selective serotonin reuptake inhibitor (SSRI). It increases concentrations of serotonin in the synaptic cleft by inhibiting it's reuptake.
Answer 3: Tramadol acts as a mu-opioid receptor agonist.
Answer 5: Linezolid is a bacteriostatic antibiotic medication that acts by disrupting translation of mRNA
-
A researcher experimenting with limb patterning removes some tissue from 1 part of the limb bud (which we shall call Site A) and transplants it along the anteroposterior (AP) axis to create a mirror- hand duplication. Which of the following is true?
-
Site A is the apical ectodermal ridge (AER). Site A tissue expresses Shh protein.
-
Site A is the AER. Site A tissue expresses FGF8.
-
Site A is the zone of polarizing activity (ZPA). Site A tissue expresses Shh protein.
-
Site A is the zone of polarizing activity (ZPA). Site A tissue expresses FGF8 protein.
-
Site A is non-AER ectoderm. Site A tissue expresses WNT7a.
CORRECT ANSWER: 3
The ZPA is located on the posterior (ulnar) margin of the limb bud. It expresses Shh protein. When tissue from ZPA is added to the anterior (radial) margin of the limb bud, ulnar dimelia, or mirror hand duplication, occurs.
The ZPA controls AP (radioulnar) growth. The signaling molecule is Shh, which is dose dependent. Higher Shh doses lead to posterior (ulnar) digits ulnar
sided polydactyly. The extent of duplication is dose dependent (higher dose = more replication). Reduced Shh leads to loss of digits. Posterior elements (little finger/ulna) are formed EARLY prior to anterior elements which are formed LATE (radius/thumb).
Disruption of AP patterning will result in loss of later forming elements (radius/thumb).
Al-Qattan et al. reviewed embryology of the upper limb. They summarized that embryology of the upper limb can be viewed in 2 distinct ways: the steps of limb development and the way that the limb is patterned along its 3 spatial axes. Cell signaling plays a major role in regulating growth and patterning of the vertebrate limbs. Signaling cell dysfunction results in congenital
differences according to the affected signaling axis.
Illustration A shows an experiment to create ulnar dimelia by adding ZPA tissue to the anterior limb bud. The video shows development of the limb.
Incorrect Answers:
Answers 1 and 2: Site A is the ZPA. The AER controls proximal to distal patterning.
Answer 4: The ZPA expresses Shh. The AER expresses FGF8.
Answer 5: Non-AER ectoderm controls dorsoventral patterning. Dorsal tissue express WNT7a and ventral tissue express en-1 protein.
-
A 71-year-old caucasian woman was the passenger in a low speed motor vehicle accident. She complained of acute back pain and was evaluated in the emergency department. On exam, she has point tenderness about her lower thoracic region and has no neurologic symptoms. Radiographs were obtained (Figures A). Which of the
following is true about this patient's condition?
-
Her T score is likely -2.0
-
Vertebroplasty has not been shown to be beneficial for the treatment of this injury in randomized controlled trials
-
A CT scan is needed for further evaluation the patient's condition
-
Distal radius fractures are a more common fragility fracture than the patient's current injury
-
The patient's injury is most likely secondary to an underlying neoplastic condition
CORRECT ANSWER: 2
The patient in the above question has underlying osteoporosis and has suffered from a vertebral compression fracture (VCF). In randomized controlled trials (RCT), vertebroplasty has been shown to be of no benefit in the treatment of VCFs.
Osteoporosis affects more than 12 million Americans per year, with the burden falling heaviest on postmenopausal women. Because of decreased bone strength, patients with osteoporosis are susceptible to fragility fractures. With no additional risk factors, a 65- year-old white woman has a 10% 10-year risk of a fragility fracture. VCFs are the most common type of fragility fracture. Though most of these fractures will heal, many patients will have residual pain and disability despite conservative therapy. Vertebroplasty was once considered a good surgical method of treating these patients. However, vertebroplasty has been shown in RTCs to have limited efficacy and is no
longer considered an indicated procedure in this clinical scenario. If the patient continues to have pain, other options such as kyphoplasty are considered.
Buchbinder et al. investigated the efficacy of pain reduction by vertebroplasty in patients with painful osteoporotic vertebral fractures. They performed a double-blinded, randomized control trial with 78 patients comparing vertebroplasty with sham surgery and found that there was no statistically significant reduction in pain between groups at 1 week, 1 month, 3 months, or
6 months. The authors concluded that vertebroplasty provided no beneficial effect in patients with painful osteoporotic vertebral fractures and they questioned its utility as a treatment for these patients.
The U.S. Preventive Services Task Force (USPSTF) updated their guidelines and recommendations about screening for osteoporosis in both men and women. They discussed current screening methods and tools for predicting risk of osteoporotic fractures, and medical management to decrease the risk for osteoporotic fractures. The update is different from the 2002 recommendation as they address screening for osteoporosis in men and they recommended screening all women whose 10-year fracture risk is equal to or
greater than that of a 65-year-old white woman with no additional risk factors.
Figure A is a lateral radiograph of a vertebral compression fracture at the level of L4 with height loss and anterior wedging.
Incorrect Answers:
Answer 1: Her T score will likely be below -2.5.
Answer 3: A CT scan in a neurologically intact patient with adequate plain films is not indicated.
Answer 4: The order of fragility fractures are as follows from most common to least- VCFs> hips> distal radius
Answer 5: The patient's cause of the VCF is mostly likely osteoporosis with this demographic and mechanism.
-
Which of the following treatments for osteoporosis is a direct inhibitor of RANK ligand (RANK-L)?
-
Romosozumab
-
Zoledronic acid
-
Denosumab
-
Teriparatide
-
Blosozumab
CORRECT ANSWER: 3
Prolia, or denosumab, is a newly approved drug used to treat osteoporosis and has a mechanism of action similar to osteoprotegerin (inhibits binding of
RANKL to RANK).
RANKL (Receptor activator of nuclear factor kappa-B ligand) is a key molecule for osteoclast differentiation and activation. Inhibition of RANKL activity with anti-RANKL
antibody reduces osteoclastogenesis, resulting in inhibition of bone resorption.
Capozzi et al. author a review article on denosumab. They state the medication confers improved bone mineral density and prevents new fragility fractures similar to alendronate. However, denosumab presents less risk of atypical femoral fractures and osteonecrosis of the jaw.
Yasuda et al. present a review that details the creation of three elegant animal models to mimic metabolic bone disease and how the animal models can
create a template to help cure human metabolic bone disease. These enable modeling of osteoporosis, hypercalcemia, and osteopetrosis by treating normal mice with soluble RANKL (sRANKL), adenovirus expressing sRANKL, and anti- mouse RANKL neutralizing antibody, respectively. They report that these animal models can be established in about 14 days using normal mice.
Illustration A demonstrates the mechanism of action of bisphosphonates and denosumab.
Incorrect Answers:
1: Romosozumab is the first humanized anti-sclerostin monoclonal antibody that has been demonstrated to increase bone formation.
2: Zoledronic acid (Reclast) is a nitrogen containing bisphosphonates that inhibits osteoclast resorption by inhibiting the enzyme farnesyl diphosphate synthase.
4: Teriparatide (Forteo) comprises the first 34 amino acids of the 84 amino acid parathyroid hormone (PTH) and can reproduce the primary effects of PTH by activating adenyl cyclase.
5: Blosozumab is an investigational monoclonal anti-sclerostin antibody showing osteoanabolic properties with the potential to improve clinical
outcomes in patients with osteoporosis.
-
Which of the following functions by inhibiting osteoblastogenesis in order to decrease bone formation?
-
RANKL
-
Interleukin 1 (IL-1)
-
WNT signaling
-
Sclerostin
-
Osteoprotegrin
CORRECT ANSWER: 4
Sclerostin is a negative regulator of bone formation by inhibiting osteoblastogenesis.
Sclerostin is a glycoprotein expressed by osteocytes as a potent regulator of bone formation. Sclerostin impedes osteoblast proliferation and function by inhibiting the Wnt signaling pathways and thus inhibits bone formation.
Suen et al. performed a lab experiment with 120 rats that underwent femur fractures. The subjects were given Sclerostin monoclonal antibody (Scl-Ab) treatment or placebo. The Scl-Ab treatment group showed showed significantly higher ultimate load during mechanical testing and increased fracture healing with histologic testing. They concluded that an antibody to inhibit sclerostin
will have a net positive effect on fracture healing by allowing osteoblastogenesis.
Illustration A depicts how inhibition of sclerostin stimulates bone formation, and antibodies to sclerostin are potentially beneficial in the management of osteoporosis and some other skeletal diseases.
Incorrect Answers:
1: RANKL is secreted by osteoblasts and binds to the RANK receptor on osteoclast precursor and mature osteoclast cells
2: Interleukin 1 (IL-1) stimulates osteoclast differentiation and thus bone resorption 3: WNT signaling promotes osteoblast formation. In addition to increasing osteoblast
formation, the pathway upregulates osteoprotegrin (OPG) and OPG works by blocking osteoclastogenesis.
5: Osteoprotegrin is a decoy receptor produced by osteoblasts and stromal cells that binds to and sequesters RANKL and thus inhibits osteoclast
differentiation, fusion, and activation.
-
Which of the following most accurately describes stainless steel?
-
Composed of iron-carbon alloy, modulus of elasticity less stiff than bone
-
Composed of cobalt-chrome-molybdenum alloy, modulus of elasticity more stiff than bone
-
Composed of iron-carbon alloy, modulus of elasticity more stiff than titanium
-
Composed of cobalt-chrome-molybedenum alloy, modulus of elasticity less stiff than titanium
-
Composed of iron-carbon alloy, modulus of elasticity is more stiff than bone, cobalt- chrome, and aluminum-oxide (ceramic)
CORRECT ANSWER: 3
Stainless steel is primarily an iron-carbon alloy with other elements including molybdenum, chromium, and manganese. Illustration A demonstrates Young's modulus of elasticity for multiple orthopaedic biomaterials. Stainless steel is stiffer than bone and titanium but less stiff than ceramics and cobalt-chrome. Titanium most closely emulates the modulus of elasticy of bone. Friedman, et al reviews the basic sciences of orthopaedic biomaterials.
-
A 60-year-old man undergoes bilateral total hip arthroplasty under a single anesthetic. Which intervention will best reduce postoperative anemia and the need for allogeneic blood transfusion?
-
Normotensive epidural anesthesia
-
Spinal anesthesia
-
Hypotensive epidural anesthesia
-
Hypotensive total intravenous anesthesia
-
General anesthesia
CORRECT ANSWER: 3
In this patient, hypotensive epidural anesthesia (HEA) will best reduce postoperative anemia and the need for transfusion.
HEA creates an epidural block up to T2 to block cardio-acceleratory fibers of the thoracic sympathetic chain, and allow intraoperative mean arterial pressure (MAP) of 50mmHg.
Resultant bradycardia (and excessive hypotension) is treated with low-dose intravenous epinephrine. Other measures to lower the transfusion trigger include preoperative erythropoietin (EPO), preoperative autologous blood donation (ABD), and postoperative cell saving. Preoperative ABD can be used with acute normovolemic hemodilution (ANH) where 2-3 units of blood is exchanged for crystalloid/colloid during surgery, and retransfused back after surgery.
Moonen et al. reviewed perioperative blood management. For EPO, they state that 600 IE/kg weekly starting 3 weeks prior, or 300IE/kg daily for 14 days prior. They also recommend COX2 inhibitors to avoid the COX1-mediated antiplatelet effects of NSAIDS.
Morris et al. compared bipolar vs standard monopolar electrocautery in anterior THA. They found similar transfusion rates, postoperative hemoglobin and postoperative drop in hemoglobin between groups, although the bipolar group had average of 12.5cc more blood loss (140cc vs 127.5cc, p=0.03). They concluded that the utilization of hypotensive anesthesia explained the low blood loss. They discontinued routine use of bipolar cautery with anterior THA.
Incorrect Answers
Answers 1: Hypotension (reduced MAP of 50mmHg) is necessary to reduce intraoperative bleeding.
Answer 2: HEA results in reduced blood loss (because of hypotension) and postoperative transfusion compared with spinal anesthesia
Answer 4: Although hypotensive total intravenous anesthesia produces similar MAP levels compared with HEA, HEA results in less intraoperative blood loss. It is hypothesized that this is because HEA is associated with non-positive pressure ventilation (unlike HTIVA), better distribution of blood flow, and lower mean intraoperative central venous pressure.
Answer 5: Unlike general anesthesia, regional anesthesia (epidural and spinal) results in lower arterial blood pressure, lower central venous pressure, and lower peripheral venous blood pressure, resulting in lower blood loss.
-
A researcher studies growth factors that have positive effects on cartilage healing. In vivo and in vitro experiments are performed with Growth Factor A. The properties of Growth Factor A include (1) it is the most widely investigated growth factor in cartilage repair, (2) it
increases extracellular matrix synthesis in cartilage and mesenchymal stem cells, and (3) it also triggers synovial proliferation and fibrosis. Which of the following is most likely to be Growth Factor A?
-
Interleukin-1 (IL-1)
-
Tumor necrosis factor-alpha (TNF-alpha)
-
Fibroblast growth factor (FGF)
-
Transforming growth factor-beta 1 (TGF-beta1)
-
Platelet-derived growth factor (PDGF)
CORRECT ANSWER: 4
TGF-beta 1 stimulates the synthesis of extracellular matrix (ECM) and causes synovial proliferation and fibrosis.
TGF-beta is the most thoroughly investigated member of the TGF-beta superfamily. This group includes TGF-beta1, BMP-2, and BMP-7. Besides the above activities, TGF-beta1 also stimulates chondrocyte synthetic activity and decreases the catabolic activity of IL-1.
Fortier et al. reviewed the role of growth factors in cartilage repair and modification of osteoarthritis. They found that members of the TGF-beta superfamily, FGF family, IGF1, and PDGF have all been investigated as possible treatment augments in the management of chondral injuries and early arthritis. They concluded that more research was necessary before routine application.
Illustration A shows a summary of the different growth factors and their effects on cartilage.
Incorrect Answers:
Answer 1: IL-1 is catabolic and leads to cartilage breakdown rather than synthesis. Answer 2: TNF-alpha is a proinflammatory cytokine. It does not lead to cartilage synthesis.
Answer 3: Although FGF-2 increases proteoglycan synthesis and cell proliferation, it also increases inflammation and osteophyte formation and does not aid in healing of cartilage defects. FGF-18 is less well studied. Answer 5: PDGF has no adverse or positive effects on cartilage when used in vivo. Much evidence on its role is extrapolated from the effects of PRP.
-
Passage of a sodium ion through a voltage-gated channel leads to which of the following?
-
Apoptosis of gram negative bacteria
-
Binding of RANKL to osteoblasts
-
Inhibition of micturition
-
Generation of a nerve action potential
-
Deposition of salts in adipose tissue
CORRECT ANSWER: 4
Passage of sodium through a voltage-gated channel will lead to generation of a nerve action potential.
Voltage-gated channel are shut when the membrane potential is near the resting potential of the cell, but they rapidly begin to open if the membrane potential increases to a precisely defined threshold value. When the channels open (in response to depolarization in transmembrane voltage), they allow an inward flow of sodium ions, which changes the electrochemical gradient, which in turn produces a further rise in the membrane potential. This then causes more channels to open, producing a greater electric current across the cell membrane, and so on.
Lee et al. present a review article on nerve conduction and needle electromyography studies. They note that the three types of nerve conduction study are motor, sensory, and mixed, of which motor is the least sensitive. In
addition, they report that peripheral nerve entrapment initially results in focal demyelination; thus, nerve conduction velocity slows across the site. However, with radiculopathy and nerve root compression, the nerve conduction study may be normal.
Catterall presents a review article covering an overview of structural models of voltage- dependent activation, sodium selectivity and conductance, drug block and both fast and slow inactivation. He notes that voltage-gated sodium channels initiate action potentials in nerve, muscle and other excitable cells.
Illustration A is a diagram that shows the electrical recordings of an action potential, along with labels of each section of the process.
-
Metastatic disease of several cancers create lytic lesions because these cancers:
-
directly produce osteoprotegerin
-
stimulate osteoblasts to produce osteoprotegerin
-
directly produce receptor activator of nuclear factor kappa beta ligand (RANKL)
-
stimulate osteoclasts to produce receptor activator of nuclear factor kappa beta ligand (RANKL)
-
directly produce an analog to calcitonin Correct answer: 3
Several cancers directly produce receptor activator of nuclear factor kappa beta ligand (RANKL) which leads to bone resorption and the lytic lesions seen on radiographs.
Cancers that originate or metastasize to bone can be blastic, lytic, or mixed when viewed on radiographs. Lytic lesions appear radiolucent due to bone resorption that occurs around the cancer cells. This is secondary to direct production and release of RANKL by the cancer cells themselves. RANKL then stimulates osteoclastogenesis and an increase in local bone resorption. Denosumab is a monoclonal antibody against RANKL that has been shown to decrease rates of pathologic fractures.
Lynch et al. review the progression of metastatic disease. They note that cancer cells can stimulate both osteogenesis and osteolysis, and that this pathologic increase in bone matrix turnover is what allows the cancer to progressively expand or metastasize to distant sites. They discuss the role of matrix metalloproteinases as primary regulators of this process.
Illustration A is a diagram illustrating how the binding of RANKL to receptors on the surface of osteoclast precursors activates them and begins the process of bone resorption. Cancer cells can directly produce RANKL thus increasing the rate of local bone resorption.
Incorrect Answers:
Answers 1 and 2- Osteoprotegerin acts by sequestering RANKL as a decoy receptor and causes a decrease in bone resorption.
Answer 4- Osteoclasts do not secrete RANK-ligand, rather they express the receptor (RANK) for the ligand.
Answer 5- Calcitonin acts directly on osteoclasts to inhibit bone resorption.
-
A postmenopausal patient comes to your office for follow-up after a dual- energy x-ray absorptiometry (DEXA) test. The T-score is
-0.7SD. The Z score is -0.4SD. By World Health Organization (WHO) criteria, these DEXA findings would merit a diagnosis of
-
Normal
-
Osteopenia based on Z-score findings
-
Osteopenia based on T-score findings
-
Osteopenia based on T- and Z-score findings
-
Osteoporosis
CORRECT ANSWER: 1
By WHO classification, her findings (T-score of -0.7SD) would merit a diagnosis of Normal bone mineral density (BMD).
The WHO classifies bone density in postmenopausal women based on T- scores. Patients are classified based on the lowest T score of the spine, femoral neck, trochanter, or total hip. The classification should not be used with peripheral measurements. Z-scores are used for premenopausal women,
younger men, and in children. The Z score compares a patient with age-, sex-, and race- matched norms.
Blake and Fogelman reviewed the role of central DEXA in treatment of osteoporosis, compared with quantitative CT, peripheral DEXA and quantitative ultrasound. The advantages of central DEXA include results that can be interpreted using WHO T-score definitions, ability to predict fracture risk, and effectiveness at targeting antifracture treatments.
Templeton reviewed secondary osteoporosis. In women with osteoporosis, the most common causes of secondary osteoporosis include hypercalciuria, malabsorption, hyperparathyroidism, vitamin D deficiency, and exogenous hyperthyroidism. In men with osteoporosis, the most common causes of secondary osteoporosis include hypogonadism, corticosteroid use and alcoholism.
Unnanuntana et al. reviewed the assessment of fracture risk. The FRAX (World Health Organization Fracture Risk Assessment Tool) calculates 10-year risk of fracture based on the following variables: age, sex, race, height, weight, BMI, history of fragility fracture, parental history of hip fracture, use of oral glucocorticoids, secondary osteoporosis and alcohol use to calculate 10-year risk of fracture. They also discussed biochemical markers of bone formation
and resorption, which are useful for monitoring the efficacy of antiresorptive / anabolic therapy, and may help identify patients at high risk for fracture.
Illustration A shows the WHO classification. Illustration B shows a comparison between central DEXA and other methods of BMD measurement.
Incorrect Answers:
Answers 2, 3, 4: In a postmenopausal woman, only the T-score is used for classification. Osteopenia is defined as T score of between 1 and 2.5 standard deviations below the norm.
Answer 5: Osteoporosis is defined as a T-score of more than 2.5 standard deviations below the norm.
-
You are staffing the prison clinic in a large public hospital when a 55-year-old African American male presents complaining of severe
right hip pain. His pain has been ongoing for the past five years and limits his ambulation. He has never used medications for pain control or physical therapy. A radiograph is shown in figure A. When formulating his treatment plan, it is important to:
-
Guarantee the success of total hip arthroplasty
-
Recommend simultaneous bilateral total hip arthroplasty
-
Understand the role of implicit bias as a determinant of health care delivery disparity
-
Request the patient reveal the reason for his incarceration
-
Suggest referral to a pain management clinic
CORRECT ANSWER: 3
The patient is an African American male prisoner with symptomatic right hip osteoarthritis. When formulating a treatment plan, it is important to
understand the role of physician implicit bias in delivery of care and in creating disparities in healthcare delivery.
Physician bias, prejudice, discrimination, and clinical uncertainty are all factors that contribute to health care disparities in the United States. Implicit and explicit attitudes are cognitive traits that influence physician delivery of care, and sometimes these attitudes do not perfectly correspond. It is important for a physician to understand that his implicit attitudes about a patient may unintentionally influence care despite his explicit attitudes.
Physicians should be aware of their implicit biases in order to provide more effective decision- making and quality of care.
Stone et al. write about the issue of culturally competent delivery of care and
the avoidance of unconscious bias in medical decision making. They argue that because unconscious stereotypes and prejudices can trigger biased medical decisions against specific groups, leading to the creation of differential diagnoses, disparities in treatment, and causing minorities to feel uncomfortable with seeking or complying with treatment plans. The authors suggest the integration of cultural competency training into medical education in order to help understand the perspective of the minority group patient.
Sabin et. al. compared the implicit and explicit biases of physicians with respect to race, gender, and age. They found that medical doctors showed an implicit bias of preferentially caring for White Americans relative to Black Americans, independent of the doctors’ self- report (explicit biases). Doctors'implicit biases exceeded their explicit biases in all race groups studied, except for African American physicians, who did not show an implicit bias toward patients.
Figure A demonstrates an AP pelvis x-ray with severe arthrosis of the right hip. The left hip demonstrates moderate disease.
Incorrect Answers:
Answer 1: While total hip arthroplasty has a high rate of success for osteoarthritis, it is important to avoid making guarantees of success. Answer 2: The patient has symptomatic right hip osteoarthritis.
Radiographically he has severe right hip osteoarthritis with mild to moderate disease in the left hip. Left hip arthroplasty is not indicated in an asymptomatic hip.
Answer 4: Requesting that the patient reveal the reason for incarceration may reveal or contribute to the physician's implicit biases. Care should be taken during this conversation in order to uphold a high quality of care.
Answer 5: The patient has known osteoarthritis, which has a surgical solution as well as several non-operative modalities that have not yet been attempted. Referral to pain management at this point in the discussion is not appropriate at this time.
-
Which of the following genetic disorders has an X-linked recessive inheritance pattern?
-
Gaucher disease
-
Prader-Willi Syndrome
-
Diastrophic Dysplasia
-
Hemophilia A
-
Hypophosphatemic rickets
CORRECT ANSWER: 4
Hemophilia A is inherited in an X-linked recessive fashion.
X-linked recessive disorders occur when a mutation occurs on the X chromosome that causes the phenotype to be expressed in males and in females who are homozygous for the gene mutation. These disorders are much more common in males since they only have one X chromosome. Other X-linked recessive disorders include: Duchenne muscular dystrophy, Becker's muscular dystrophy, Hunter's syndrome, and spondyloepiphyseal dysplasia (SED) tarda.
Vanderhave et al. reviewed the orthopaedic consideration in patients with hemophilia. Amongst other things, they discuss arthroplasty in patients with this condition. While they are at higher risk for stiffness and acute hemarthrosis following total knee arthroplasty, ~90-95% of patients have good or excellent results.
Illustration A shows how hemophilia is inherited through a punnett square. Illustration B shows how a weak clotting mechanism causes increased bleeding in patients with hemophilia.
Incorrect Answers:
Answer 1: Gaucher disease is an autosomal recessive condition.
Answer 2: Prader-Willi Syndrome is inherited through imprinting. Imprinting is a genetic phenomenon by which certain genes are expressed in a parent-of- origin-specific manner. Answer 3: Diastrophic dysplasia is an autosomal recessive condition. Answer 5: Hypophosphatemic rickets is a sex-linked dominant condition.
-
Which of the following best describes the process of transcription?
-
Reading DNA information to synthesize and replicate during the S phase of the cell cycle
-
Generating a haploid product with a genotype that differs from both haploid genotypes that constituted the meiotic diploid cell.
-
Generating a polypeptide whose amino acid sequence is derived from the codon sequence of an mRNA molecule.
-
Exchanging of two nonhomologous chromosomes resulting in chromosomal rearrangement mutations
-
Reading DNA information by RNA polymerase to make specific complementary mRNA
CORRECT ANSWER: 5
The process of reading DNA information by RNA polymerase to make specific complementary mRNA is known as transcription.
Transcription relies on the complementary pairing of bases to create mRNA from DNA. The two strands of the double helix separate, and one of the separated strands serves as a template. Then, free nucleotides are arranged on the DNA template by their complementary ribonucleotide bases in the template. The free ribonucleotide A aligns with T in the DNA, G with C, C with G, and U with A.
Musgrave et al discuss orthopaedic surgery related gene therapy and tissue engineering topics. Specific sectors of musculoskeletal medicine where gene therapy and tissue engineering have shown promise and early treatment success include the areas of bone healing, cartilage repair, intervertebral disk pathology, and skeletal muscle injuries.
Incorrect Answers:
1: This is the definition of replication.
-
This is the definition of recombination. Recombinants are those products of meiosis with allelic combinations different from those of the haploid cells that formed the meiotic diploid.
-
This is the definition of translation.
-
This is the definition of translocation. Examples of translocation include Ewing's sarcoma, Rhabdomyosarcoma, and Synovial sarcoma
-
In the treatment of rheumatoid arthritis, which medication is an antagonist of tumor necrosis factor-alpha?
-
Rituximab
-
Etanercept
-
Abatacept
-
Methotrexate
-
Leflunomide
CORRECT ANSWER: 2
Etanercept is a biochemically designed tumor necrosis factor receptor immunoglobulin G fusion protein, which binds to TNF-alpha and is thus a TNF- alpha antagonist.
TNF-alpha is considered to be one of the major cytokines involved in rheumatoid arthritis pathology. As a result, many biologic agents used to treat rheumatoid arthritis (RA) are manufactured to block TNF-alpha or its
receptors. This has been shown to reduce inflammation and stop disease progression. In the USA, Etanercept is approved to treat rheumatoid arthritis, juvenile rheumatoid arthritis and psoriatic arthritis, plaque psoriasis and ankylosing spondylitis. The route of administration is subcutaneous.
Bongartz et al. used a randomized control trial to asses the risk of infection and
malignancy rates in RA treated with TNF-alpha antagonist. Overall, patients with RA appear to have an approximately 2-fold increased risk of serious infection compared to the general population and non-RA controls, irrespective of TNF-alpha antagonist use. The pooled odds ratio for malignancy was 3.3 (95% confidence interval [CI], 1.2-9.1) and for serious infection was
2.0 (95% CI, 1.3-3.1) with use of TNF-alpha antagonist.
Howe et al. review the medical management of patients with RA who underwent orthopaedic procedures. They state that while there is conflicting information regarding TNF-alpha antagonists, they recommend holding them prior to major orthopaedic interventions.
Incorrect Answers:
Answer 1: Rituximab is a monoclonal antibody to CD20 antigen (inhibits B cells). It is often used with good clinical outcomes as monotherapy in patients who are intolerant of methotrexate or have contraindications to methotrexate or other DMARDs.
Answer 3: Abatacept is a selective costimulation modulator that binds to CD80 and CD86 (inhibits T cells). It is often prescribed for treatment of moderate to severe rheumatoid arthritis, or after failure of a disease-modifying anti- rheumatic agent (DMARD), like methotrexate but it can be used as first-line therapy.
Answer 4: Methotrexate is a folic acid analogue. It binds dihydrofolate reductase and prevents synthesis of tetrahydrofolate. It is usually a first line treatment for moderate to severe rheumatoid arthritis.
Answer 5: Leflunomide is an inhibitor of pyrimidine synthesis. It is approved to treat adult moderate to severe rheumatoid arthritis, usually as a monotherapy or failure of other DMARDs.
-
Cortical bone demonstrates viscoelastic behavior as its mechanical properties are sensitive to strain rate and duration of applied load. Regarding longitudinal strain in cortical bone, which of the following statements regarding this characteristic is true?
-
As strain rate increases, both elastic modulus and ultimate strength increase
-
As strain rate increases, elastic modulus remains unchanged but ultimate strength increases
-
As strain rate increases, elastic modulus increases but ultimate strength decreases
-
As strain rate increases, both elastic modulus and ultimate strength decrease
-
As strain rate increases, elastic modulus increases but ultimate strength remains unchanged
CORRECT ANSWER: 1
As strain rate increases, both elastic modulus and ultimate strength increase. For LOW strain rates typical of normal activity (physiological strain rates of
<0.1/s), bone is ELASTIC and DUCTILE (increasing ultimate strain with increasing strain rate). There is a ductile-to-brittle transition with increasing strain rate from normal to supranormal rates. For EXTREMELY HIGH supranormal strain rates (>0.1/s, high impact trauma), bone is VISCOELASTIC and BRITTLE (low ultimate strain with increasing strain rate). Bone also becomes stronger and stiffer (higher modulus, steeper slope of stress- strain plot) as strain rate increases. This viscoelastic property helps in damping muscle contracture.
Natali and Meroi reviewed studies examining mechanical properties of bone. Mechanical properties are correlated with moisture, deformation rate, density and region of bone.
Mechanical adaptation of bone is affected by strain rate (rate at which bone is deformed), strain mode (tension, compression, shear), strain direction (direction of strain relative to bone surface), strain frequency (cycles/second), stimulus duration (period over which deformation cycles are applied), strain distribution (pattern of strain magnitude across bone section) and strain energy (energy stored during deformation).
Illustration A shows the mechanical properties of bone with increasing strain rates. Illustration B shows that the ultimate strength and elastic modulus increase with rapid loading or deformation. The ultimate strength increases by roughly a factor of 3, while the elastic modulus increases by a factor of
approximately 2 over the strain rate range.
Incorrect Answers:
Answers 2, 3, 4, 5: As strain rate increases, elastic modulus and ultimate strength increase. During normal activity, as strain rate increases, bone is more ductile. With high impact trauma, bone is more brittle.
-
In regards to a genetic disorder, which of the following is an example of "anticipation?"
-
Gene characteristics more severe and earlier in onset in subsequent generations
-
A disorder inherited from a genetic mutation specific to maternal DNA
-
Gene characteristics expressed to varying degrees in different individuals
-
Variation in the relative frequency of a genotype due to chance
-
The presence of an extra copy of a chromosome
CORRECT ANSWER: 1
Genetic anticipation is a phenomenon in which a genetic disorder becomes progressively more severe and earlier in onset with each generation. Examples of disorders exhibiting anticipation include Huntington's disease and myotonic dystrophy.
Genetic anticipation is an important concept in understanding the development and genetic implications of many heritable disorders. It is a common phenomenon in trinucleotide repeat expansion disorders. These disorders are due to unstable microsatellite trinucleotide repeats that expand beyond the normal threshold. In subsequent generations these expansions become longer and thus express disease characteristics at a younger age of onset, and often with greater severity.
Martorell et al. investigated the development of CTG trinucleotide repeats in patients with myotonic dystrophy type 1 (DM1) and their relatives. They discovered unaffected individuals carry a pre-mutation sequence which can lead to trinucleotide repeat expansion in subsequent generations and thus produce offspring with the disorder.
Kamsteeg et al. compare the characteristics of DM1 and DM2. Both are due to trinucleotide repeat expansions. However, while DM1 can present with earlier onset and increasing severity in each generation, DM2 does not exhibit this genetic anticipation.
Incorrect Answers
Answer 2: "Genomic imprinting" is when a disorder is linked to a parent- specific origin. An example of maternal genomic imprinting is Angelman Syndrome. An example of paternal genomic imprinting is Prader Willi.
Answer 3: "Variable penetrance" is when gene characteristics are expressed in varying degrees.
Answer 4: "Genetic drift" is the chance variation in the relative frequency of a genotype within a population.
Answer 5: "Trisomy" is the presence of an extra copy of a chromosome. Down Syndrome is trisomy 21, which is due to an extra copy of chromosome 21.
-
A researcher is working on Medication A, a drug FDA-approved for the treatment of osteoporosis in men and women. It is an anti- resorptive agent that inhibits the formation, function and survival of osteoclasts. It does not bind to calcium hydroxyapatite. At 1-year after the initial dose, tissue levels are non- detectable. It can be used in the presence of cancer metastases to bone. What is Medication A?
-
Denosumab
-
Alendronate
-
Abaloparatide
-
Teriparatide
-
Strontium ranelate
CORRECT ANSWER: 1
Denosumab is FDA-approved for the treatment of osteoporosis in men and women. It inhibits the formation, function and survival of osteoclasts (OC). It does not bind to calcium hydroxyapatite. At 1-year after the initial dose, tissue levels are non-detectable.
Denosumab is a human monoclonal antibody against RANKL. By binding RANKL, it prevents interaction of RANKL with RANK (on OC and osteoclast precursors, OCP), and inhibits OC-mediated bone resorption, and the formation, function and survival of OC. In contrast, bisphosphonates bind to calcium hydroxyapatite in bone, and decrease resorption by decreasing function and survival (but not formation) of OC.
Vaananen et al. reviewed the cell biology of OC. During bone resorption, 3 membrane domains appear: ruffled border, sealing zone and functional secretory domain. The resorption cycle starts with migration, bone attachment, polarization (formation of membrane domains), dissolution of hydroxyapatite, degradation of organic matrix, removal of degradation
products from resorption lacuna, and apoptosis of the OC or return to the non- resorbing stage.
Boyce et al. reviewed the regulation of osteoclasts and their functions. OCPs are held in bone marrow by chemokines e.g. stroma-derived factor-1 (SDF1) and attracted to blood by sphingosine-1 phosphate (S1P) (increased in synovial fluid of patients with RA). All aspects of osteoclast formation and functions are regulated by M-CSF and RANKL. More recent studies indicate that osteoclasts and their precursors regulate immune
responses and
osteoblast formation and functions by means of direct cell-cell contact through ligands and receptors, such as ephrins and Ephs, and semaphorins and
plexins, and through expression of clastokines.
Warriner and Saag reviewed the diagnosis and treatment of osteoporosis. They defined osteoporosis as T-score of = -2.5 or a history of fragility fracture. Incident hip and vertebral fractures increase future risk of these fractures (hazard ratio 7.3 and 3.5, respectively).
Cummings et al. compared subcutaneous denosumab (60mg every 6mths) vs placebo in prevention of fractures in 7868 osteoporotic (T-score -2.5 to -4.0) postmenopausal women. They found that denosumab reduced risk of vertebral fracture by 68% (risk ratio, 0.32), hip fracture by 40% (hazard ratio 0.6), nonvertebral fracture by 20% (hazard ratio 0.8). There was no increased risk of cancer, infection, delayed fracture healing, cardiovascular disease, osteonecrosis of the jaw or adverse reactions. They concluded that it was useful for reduction of fractures in osteoporotic women.
The video shows the action of denosumab (prolia). Illustration A shows the different osteoclast zones.
Incorrect Answers:
Answers 2: Alendronate (and other bisphosphnates) inhibit resorption of bone, decrease function and survival of osteoclasts. Because of binding to calcium hydroxyapatite, they are detectable years after dosing. They reduce function and survival of OC, but do not affect the formation of osteoclasts.
Answer 3: Abaloparatide is a PTH analog that has completed phase III trials for osteoporosis. As of mid-2016, it is not yet approved for treatment of osteoporosis. Answer 4: Teriparatide (recombinant PTH 1-34) is the only anabolic (not antiresorptive) agent approved for osteoporosis treatment. It is administered by daily subcutaneous injection. Osteosarcoma, cancer metastases to bone and Paget's disease are contraindications.
Answer 5: Strontium ranelate (marketed as Protelos or Protos) both increases deposition of new bone by osteoblasts and reduces the resorption of bone by osteoclasts ("dual action bone agent", DABA). It is not FDA approved for use in the United States. Increased risk of myocardial infarction has been detected.
-
Which specific legislative Act in the United States was created to require reporting of annual monetary gifts or compensation of more than $10 by orthopaedic implant companies to physicians?
-
Patient Protection and Affordable Care Act
-
Medicare Payment Reform Act
-
Physician Financial Transparency Act
-
Physician Payments Sunshine Act
-
Health Insurance Portability and Accountability Act
CORRECT ANSWER: 4
The Physician Payments Sunshine Act requires all payments by corporations to physicians beyond $10 per year to be reported to the Centers for Medicare and Medicaid Services.
Under this Act, all manufacturers of drugs and devices covered under Medicare, Medicaid, and SCHIP are obliged to federally report payments beyond $10 annually to physicians and academic centers. The Act was first introduced in 2007, enacted in 2010, and in 2014 the first data (from 2012) was reported publicly online in the Open Payment Program of the Centers for Medicare and Medicaid Services website.
Samuel et al analyze orthopedic surgeons available data from the Sunshine Act regarding industry payments and find over 110 million USD paid to approximately 15,000 orthopedic surgeons over the 5-month study period. No long term data exists to determine if these payments have any affect in healthcare.
Incorrect Answers:
Answers 1: The Patient Protection and Affordable Care Act (PPACA), known also by its shorter name of the Affordable Care Act (ACA) or it's nickname
"Obamacare", was passed in March 2010. The Sunshine Act was one of many provisions passed within the PPACA (after the Sunshine Act failed to pass on its own in prior years), but the PPACA focused primarily on improving the quality and affordability of healthcare insurance and lowering the costs of healthcare.
Answer 2: The Medicare Payment Reform Act of 1983 was a quickly drafted revision to the way Medicare payments were made, changing from fee-for- service to prospective payments allowing Medicare to determine payment amount rather than providers/hospitals.
Answer 3: This is a fictitious act.
Answer 5: HIPPA is the 1996 legislation defining standards and protections for patient private health information and electronic exchange of records.
-
Which of the following materials best approximates the Young's modulus of elasticity of cortical bone?
-
Titanium
-
Cobalt-chrome alloy
-
Alumina
-
Zirconia
-
Stainless steel
CORRECT ANSWER: 1
Of the materials listed titanium (100GPa) has an elastic modulus closest to cortical bone (approximately 18GPa) as well as cancellous bone (approximately 2GPa).
Titanium is a material that is light, highly ductile, strong and corrosion resistant. However, titanium has poor wear resistance and is notch sensitive. It is commonly used as an orthopaedic implant materials because it has torsional and axial stiffness (moduli) that most closely mimics bone. Young’s modulus is constant and different for each material and represents the material's ability to maintain shape under external loading.
Rho et al found that the average Young's modulus for trabecular bone measured ultrasonically and mechanically was 14.8 GPa (S.D. 1.4) and 10.4 (S.D. 3.5), respectively. The average Young's modulus of microspecimens of cortical bone measured ultrasonically and mechanically was 20.7 GPa (S.D.
1.9) and 18.6 GPa (S.D. 3.5), respectively.
Illustration A depicts a stress vs. strain curve. The slope of the line in the elastic zone represents the Young Modulus of Elasticity.
Incorrect Answers:
Answer 2: Cobalt-chrome alloy is approximately 240 GPa Answer 3: Alumina is approximately 340 GPa
Answer 4: Zirconia (Ceramic) = 248 GPa
Answer 5: Stainless steel is approximately 240 GPa
-
The difference between vitamin D-dependent rickets type I (VDDR I) and vitamin D-dependent rickets type II (VDDR II) is
-
VDDR I is caused by an inactivating mutation of the receptor for 1,25 (OH)2 vitamin D3. VDDR II is a deficiency of an enzyme predominantly found in the kidney.
-
VDDR I is caused by an activating mutation of the receptor for 1,25 (OH)2 vitamin D3. VDDR II is a deficiency of an enzyme predominantly found in the kidney.
-
VDDR I is a deficiency of an enzyme predominantly found in the kidney. VDDR II is caused by an inactivating mutation of the receptor for 1,25 (OH)2 vitamin D3.
-
VDDR I is a deficiency of an enzyme predominantly found in the kidney. VDDR II is caused by an activating mutation of the receptor for 1,25 (OH)2 vitamin D3.
-
VDDR I is a deficiency of an enzyme predominantly found in the liver. VDDR II is caused by an inactivating mutation of the receptor for 1,25 (OH)2 vitamin D3.
Correct answer: 3
VDDR I is a deficiency of an enzyme predominantly found in the kidney. VDDR II is caused by an inactivating mutation of the receptor for 1,25 (OH)2 vitamin D3.
VDDR I is a deficiency of 1a-hydroxylase [converts 25(OH)D to
1a,25(OH)2D3]. Lab tests show hypocalcemia, secondary hyperparathyroidism, elevated alkaline phosphatase (ALP) and low or undetectable calcitriol in the presence of adequate 25(OH)D levels. VDDR II or hereditary vitamin D resistant rickets (HVDRR) (autosomal recessive) is an inactivating mutation in the vitamin D receptor (VDR). Lab tests show low serum calcium and phosphate, elevated ALP and secondary hyperparathyroidism. Serum 25(OH)D values are normal and the 1,25(OH)2D levels are elevated (key difference from VDDR I).
Malloy et al. reviewed genetic disorders in vitamin D action. They state that VDDR I is an inborn error of vitamin D metabolism coded by the gene CYP27B1. Children with VDDR I present with joint pain/deformity, hypotonia, muscle weakness, growth failure, and hypocalcemic seizures or fractures in early infancy. Treatment is with calcitriol or 1a-hydroxyvitamin D (NOT cholecalciferol). Children with VDDR II present with bone pain, muscle weakness, hypotonia, hypocalcemic convulsions, growth retardation, severe dental caries or teeth hypoplasia. Affected children are resistant to therapy and supra-physiologic doses of all forms of vitamin D.
Illustration A shows the differences between VDDR I and VDDR II. Incorrect Answers
Answers 1, 2, 4, 5: VDDR I is a deficiency of 1a-hydroxylase (predominantly
found in the kidney). The liver enzyme vitamin D 25-hydroxylase (found in hepatocytes) is not responsible for VDDR. VDDR II is caused by an inactivating mutation (rather than an activating mutation).
-
A 73-year-old female sustains a left hip fracture that is treated with hemiarthroplasty. She has continued pain two months after surgery, and comes to you for a second opinion. Her radiograph is shown in Figure A. Which of the following best describes your responsibility in disclosing to the patient that the pain may be from a medical error?
-
You do not need to disclose this information
-
You legally must disclose this information to the patient
-
You legally must disclose this information to the original hospital's peer review panel
-
You ethically must disclose this information to the patient
-
You ethically must disclose this information to the original surgeon
CORRECT ANSWER: 4
As a practicing orthopaedic surgeon, you ethically are required to disclose the potential impact of medical errors on patient outcome.
The orthopaedic surgeon is bound ethically but not legally to give his or her best medical opinion, regardless of whether the orthopaedist is the treating physician or the physician who is asked to render a second or additional medical opinion. The best interest of the patient should clearly remain the guiding principal. It is illegal to slander the original physician if the slanderous
information is known or can be proven to be false.
Bhattacharyya et al. review the importance of documentation and ethical treatment of patients when providing second opinions. They note that it is unethical for the consulting orthopaedic surgeon to solicit care of the patient. However, at the sole discretion of the patient, the patient ethically may choose to terminate his or her relationship with his or her treating physician and then enter into another treatment relationship with the consulting
orthopaedic surgeon.
Figure A shows a left hip hemiarthroplasty with the distal component perforated through the medial proximal femur.
Incorrect Answers:
1) This information must be disclosed per ethical recommendations. 2 and 3) There is no legal requirement to disclose this information.
5) There is no documented ethical requirement to disclose this to the original surgeon.
-
A patient is consented for a right wrist open reduction and internal fixation. After the patient is prepped and draped, a skin incision is made. It is recognized intra-operatively, however, that a skin incision was made on the incorrect side (left). Subsequent right wrist open reduction and internal fixation goes uneventfully. What is the next best course of action?
-
do not tell the patient or family
-
contact the Risk Management department
-
immediately discuss the situation with the patient and family
-
alter the medical record
-
only discuss the situation with the patient if he or she brings it up. Correct answer: 3
Patients should be approached after a medical error and all errors must be promptly and completely disclosed. The physician should take the lead in the disclosure and not wait for the patient to ask. Risk management should be called as well, but the patient and family should be informed first. It is never appropriate to alter the medical record.
-
A 14-year-old female has anal hemorrhoids. The General Surgical team has asked for a consultation in regards to her history of hand, wrist, and ankle joint pain and swelling over the past 3 years. Her physical examination reveals a swollen left wrist, right knee and left ankle. Lab work shows low hemoglobin, low albumin, elevated erythrocyte sedimentation rate (ESR), elevated antinuclear antibody (ANA) count, and a negative rheumatoid factor. Radiography of the affected joints are normal. What additional work up is required prior
to her rectal surgery?
-
C-reactive protein (CRP)
-
Synovial fluid analysis of affected joints
-
Blood cultures
-
Cervical radiographs
-
Bethesda assay
CORRECT ANSWER: 4
This patient has a diagnosis of Juvenile Idiopathic Arthritis (JIA). Flexion- extension c-spine radiographs should be ordered to rule out atlantoaxial instability prior to surgery.
JIA is a persistent autoimmune inflammatory arthritis lasting more than 6 weeks in a patient younger than 16 years of age. Serologic testing for this condition will usually show elevated ESR/CRP, low hemoglobin, low albumin and an elevated anti-nuclear antibody (ANA) count, as well as negative rheumatoid factor and positive HLA-B27. Radiographs of the c-spine should be considered in patients undergoing intubation as cervical kyphosis, facet ankylosis, and atlantoaxial subluxation is associated with this condition.
Punaro et al. reviewed rheumatologic conditions in children. The typical patient with oligoarticular JIA is a white female (5:1, F:M), with a peak onset between ages 1 and 3 years. Nearly half of patients have monoarticular involvement, with the knee and ankle being most commonly involved. Uveitis is typically chronic, bilateral, and asymptomatic.
Borchers et al. reviewed juvenile idiopathic arthritis (JIA). They state that no laboratory test can conclusively establish a rheumatic diagnosis. They state that laboratory tests will be negative for systemic inflammation and antinuclear antibody (ANA) test has no use in screening for JIA, as it has a high false positive rate.
Incorrect Answers:
Answer 1: Both ESR and CRP are usually elevated in this condition and provide no further benefit for the operative management of this patient.
Answer 2: This patient has chronic joint swelling and pain. Joint aspirates and synovial fluid analysis would not be required.
Answer 3: There is no suspicion for an acute infection. Therefore, blood cultures are not required.
Answer 5: A Bethesda assay is used to measure the amount of factor VIII or IX antibody in the blood, for patients with hemophilia.
-
Induction coupling stimulates bone growth through all of the following direct effects EXCEPT:
-
Increased proliferation of osteoblasts
-
Decreased osteoclast differentiation
-
Increase release of TGF-beta1
-
Increased expression of BMP2
-
Increased expression of BMP7
CORRECT ANSWER: 2
Induction coupling stimulates bone growth by increasing expression of BMP7, BMP2, TGF-beta1, and by increasing osteoblasts proliferation. Induction coupling has not been shown to have the effect of decreasing osteoclast differentiation.
In basic science studies, electrical stimulation (i.e. induction coupling), has been shown to promote bone healing via release of growth factors that induce osteoblast differentiation/proliferation. Electrical current can be placed around bone in various ways, creating a current to stimulate growth factor release and subsequent osteoblast proliferation.
Aaron et al. summarized, in a systematic review, the effects of various types of electrical stimulation on bone and bone healing. Regardless of type (i.e. inductive coupling, capacitive coupling, direct current), they report electricity and/or electromagnetic fields promote gene expression of growth factors that promote an osteogenic environment.
Illustration A depicts a cathode placed directly to allograft with a subcutaneously placed electrical stimulator. Illustration B depicts cathodes placed anteriorly and posteriorly around the hip, connected to an outside power source to create the necessary current. Illustration C depicts an inductive coil placed laterally on the skin in order to create an electrical current.
Incorrect answers:
Answers 1,3-5: All are true effects of electrical stimulation on bone.
-
Two patients are discharged from a surgicenter after upper extremity procedures. The surgeon gives them prescriptions for oral opioid analgesics. Patient A had open reduction and internal fixation of a distal radius fracture. Patient B had cubital tunnel release without transposition. Which of the following is most likely true regarding analgesic use?
-
Patient A will use more medication than Patient B
-
Patient A will use less medication than Patient B
-
Analgesic use will be similar between Patients A and B
-
Both patients will consume more than 30 pills
-
A reasonable prescription is 40 pills with 1 refill for Patient A, and 40 pills with no refills for Patient B
CORRECT ANSWER: 1
A patient that has had ORIF will require MORE oral analgesia than a patient who has had a cubital tunnel release.
Patients undergoing bony procedures (e.g. ORIF and arthroplasty) require more analgesia than patients undergoing soft tissue procedures (e.g. carpal/cubital tunnel release, trigger finger release, elbow or shoulder arthroscopy). Overprescribing of opioid analgesia is a common problem. Many opioids are unused in the postoperative period by the patients for whom they were prescribed.
Rodgers et al. reviewed opioid use (oxycodone, hydrocodone, propoxyphene) after outpatient upper extremity surgery. Patients undergoing bony procedures used the most analgesia (14 pills) while those undergoing soft tissue
procedures used the least (9 pills). Half took medication for =< 2 days. Mean opioid consumption was 10 pills. They recommend prescribing 15 tablets with
1 refill of a Schedule III opioid analgesic for elective outpatient upper extremity procedures.
Stanek et al. proposed an educational assist device to guide pain management of postoperative hand conditions. They advocate no narcotics for trigger finger release, 10 narcotic pills for carpal tunnel, deQuervain’s, Dupuytren’s releases and small joint fusions, 20 narcotic pills for wrist ganglion cysts, hand fracture ORIF, LRTI and tendon transfers, and 40 narcotic pills for wrist fusion, open carpal surgery and DRUJ reconstruction.
Illustration A is Stanek's educational assist device for multimodal pain management of postoperative hand conditions.
Incorrect Answers:
Answers 2 and 3: Analgesic use is greater in patients undergoing bony procedures.
Answer 4: 75% of patients need =< 15 opioid analgesics, 45% took fewer than 5 pills and some took none at all.
Answer 5: A reasonable prescription is 15 pills with 1 refill of a Schedule III opioid analgesic for elective outpatient upper extremity procedures. Only 3-4% of patients will need to use the refill.
-
What property of titanium alloys accounts for their high corrosion resistance in vivo?
-
Self-passivation
-
Ductility
-
Hardness
-
Modulus of elasticity
-
Conductivity
CORRECT ANSWER: 1
In both room temperature air and physiologic fluids, titanium alloys self- passivate or spontaneously form a layer of titanium oxide very rapidly. This layer makes titanium alloys resistant to surface breakdown.
-
Embolic material generated during total knee arthroplasty (TKA) shown in Figure 29 is composed of which of the following substances?
-
Fat only
-
Fat and air
-
Fat and marrow
-
Fat and cement
-
Fat and bone
CORRECT ANSWER: 3
Emboli are created during TKA. Usually there is an increased incidence with
the use of intramedullary rods that disrupt the marrow contents. These are not fat emboli per se. They are material composed of fat cells and marrow that act like pulmonary emboli
to obstruct small arterioles in the lung. They are different from free fat emboli that are seen in fractures and that lead to chemical injury to the lung rather than obstructive injury.
-
There is increasing concern about the ethical relationship of orthopaedists to the orthopaedic equipment industry. Which of the following describes the most appropriate relationship?
-
Industry-paid travel, hotel (for the surgeon and spouse), and registration at a university- sponsored CME course
-
Industry-paid travel and hotel for a faculty member at an industry- sponsored meeting that is not CME approved
-
Consultation agreement ($50,000/annum) between the surgeon and the company for evaluation of the implant system with required oral reporting of impressions
-
A restricted grant from a company to an orthopaedic residency program with the stipulation that the third year residents be sent to an industry- sponsored course
-
Industry-paid dinner at a premium restaurant ($200/person) for surgeon and office staff at which a new set of surgical instrumentation is presented
CORRECT ANSWER: 2
It is appropriate for orthopaedic surgeons to have relationships with industry as long as the relationship is for the good of the patient and no “quid pro quo” intent exists. A grant to cover registration at a CME event is appropriate but travel and hotel for a spouse is not.
For orthopaedists who are faculty at a meeting sponsored by industry, it is appropriate for travel and expenses to be covered for that faculty member. Care must be exercised that the faculty member contributes in an amount appropriate for the expenses paid. The faculty member must ensure that information presented is unbiased and based on reasonable data and opinion. Consulting agreements should spell out specifically the duties of the agreement and payment should be appropriate for the time spent. There should be a defined work product for the consulting. Agreements that are thinly veiled payments for use of a company’s products must be avoided. In all cases, the agreements must stand up to public
scrutiny. Restricted grants for specific industry-sponsored programs aimed at residents are not appropriate. Unrestricted grants intended for attendance at approved CME courses are appropriate. Dinners at which information is presented about topics that can aid in patient care are appropriate as long as the expense is reasonable ($100 or less/person) and the guest list includes individuals who can use the information in a patient case. Clearly a “premium” dinner for office staff to review new surgical instrumentation would not pass this test.
-
Figure 33 shows the venogram of a patient who has a long history of alcohol abuse. Warfarin should be used cautiously because of the interaction with which of the following factors?
-
IV
-
V
-
VI
-
VII
-
VIII
CORRECT ANSWER: 4
Warfarin acts by inhibiting clotting factors II, VII, IX, X. The actual mechanism of action is by inhibition of hepatic enzymes, vitamin K epoxide, and perhaps vitamin K reductase.
This inhibition results in lack of carboxylation of vitamin
K-dependent proteins (II, VII, IX, X). The anticoagulant effect of warfarin can be reversed with vitamin K or fresh-frozen plasma. The use of alcohol may lead to liver dysfunction and an even more limited margin of available factors.
-
Familial (Leiden) thrombophilia is of importance in joint arthroplasty because of an abnormality in the clotting cascade. Which of the following statements best describes the condition?
-
It is a disease caused by an abnormality of platelets that leads to increased blood clotting.
-
It is a disease caused by an abnormality of vascular endothelium that leads to increased blood clotting.
-
It is a disease caused by an abnormality of hepatic metabolism that leads to decreased production of factor V and decreased blood clotting.
-
It is a disease caused by an abnormality of factor V that leads to decreased inactivation of factor Va by activated protein C (aPC) and increased blood clotting.
-
It is a familial, genetic disease that requires placement of a Greenfield filter in all
individuals who have the abnormality, prior to surgery.
CORRECT ANSWER: 4
Factor V Leiden is a disease caused by an abnormality of factor V in which a single amino acid substitution of glutamine for arginine in the protein C cleavage region leads to decreased inactivation of factor V and thus a greater tendency to form clots. More than half of all individuals with Factor V Leiden will develop deep venous thrombosis in the presence of a single additional risk factor such as long bone fracture or total joint arthroplasty.
-
Which of the following terms best describes the probability of finding a significant association in a research study when one truly exists?
-
Type-1 (alpha) error
-
Type-2 (beta) error
-
Power
-
Alpha level
-
Relative Risk
CORRECT ANSWER: 3
The power of a study is an estimate of the probability of finding a significant association in a research study when one truly exists.
The references by Kocher and Wojtys are excellent reviews of basic biostatistic principles. Incorrect Answers:
Answer 1: The power is defined by 1 - probability of type-II (beta) error, and is often set at 80%. For example, a power of 80% means that if the
intervention works, the study has an 80% chance of detecting this and a 20% chance of randomly missing it.
Answer 2: A type-II or beta error occurs when one falsely concludes that there is no significant association when there actually is an association (resulting in
a false-negative study that rejects a true alternative hypothesis). The type-II or beta error can be determined if Type I error rate and sample size are known. A type-I or alpha error occurs when a significant association is found when there is no true association (resulting in a false-positive study that rejects a true null hypothesis).
Answer 4:The alpha level refers to the probability of a type-I (alpha) error and is usually set for most studies at 0.05.
Answer 5:The relative risk is a term used in prospective cohorts studies and is the risk of developing disease for people with known exposure compared to risk of developing disease without exposure.
-
Which of the following substances makes up the majority by weight of the extracellular matrix for articular cartilage?
-
Keratin sulfate
-
Collagen type II
-
Water
-
Protein
-
Chondroitin sulfate
CORRECT ANSWER: 3
The extracellular matrix consists of water, proteoglycans, and collagen. Water makes up the majority (approximately 65% to 80%) of wet weight; 95% of the collage is type II with much smaller amounts of other collagens, including types IV, VI, IX, X, and XI. The exact functions of these other collagens are unknown, but they are believed to be important in matrix attachment and stabilization of the diameter of collagen fibrils.
-
A pediatric orthopaedic surgeon refers a child to a neurologist. The neurologist’s office requests the office records of the pediatric orthopaedic surgeon. To maintain Health Insurance Portability and Accountability Act (HIPAA) compliance, what must the surgeon obtain from the parent(s) prior to sending records?
-
No additional consent needed
-
Verbal approval
-
Written approval
-
Written approval with notarization
-
Telephone consent witnessed by a nurse
CORRECT ANSWER: 1
The privacy rules do not require an individual’s written authorization for certain permitted or required uses and disclosures of the medical records. Patient or parental authorization is not required for disclosures for certain purposes related to treatment, payment, or health care operations. Specifically, HIPAA does not require a covered entity to obtain patient authorization for many of the health care industry’s most fundamental activities such as providing care.
-
To control most spontaneous bleeding into the knee in children with hemophilia, factor VIII must be replaced to what percentage of normal?
1. 0% to 10%
2. 20% to 30%
3. 40% to 50%
4. 60% to 70%
5. 80% to 90%
CORRECT ANSWER: 3
The knee is the most common location of spontaneous bleeding in children with hemophilia. Treatment generally requires replacement to 40% to 50% of normal. For surgery, the replacement should be to 100%. The plasma level generally rises 2% for every unit (per kg body weight) of factor VIII administered.
-
What is the most important consideration in the preoperative evaluation of a child with polyarticular or systemic juvenile rheumatoid arthritis (JRA)?
-
Cervical spine assessment
-
Temporomandibular joint (TMJ)/jaw assessment
-
Dental assessment
-
Stress dosing with corticosteroids
-
Opthalmology examination
CORRECT ANSWER: 1
The cervical spine may be involved in a child with polyarticular or systemic JRA; fusion or instability can occur. Radiographic assessment of the cervical spine should include lateral flexion-extension views. The potential exists for spinal cord injury during intubation or positioning in the presence of an unstable cervical spine. Limitations of the TMJ and micrognathia may affect ease of intubation and administration of anesthesia via a mask. If the TMJ and jaw are involved, some patients may have dental findings such as dental caries and even abscesses which can affect surgery. Some children, particularly those with systemic arthritis, may be taking corticosteroids long- term and may need stress dosing with complex surgeries. Although it is important to routinely check for uveitis and iritis in children with JRA, this usually is not needed preoperatively. Uveitis and iritis are less likely in a child with systemic JRA.
-
Figure 11 shows the radiograph of a 2-year-old child with marked genu varum and tibial bowing. Based on these findings, what is the best initial course of action?
-
Obtain serum phosphorous, calcium, and alkaline phosphatase levels.
-
Obtain a scanogram to assess for limb-length discrepancy.
-
Perform bilateral valgus osteotomies to correct the deformities.
-
Measure the child for a varus prevention orthosis.
-
Educate the family about physiologic genu varum and conduct a follow-up examination in 6 months.
CORRECT ANSWER: 1
The radiograph shows multiple wide physes, consistent with a diagnosis of rickets. A low serum phosphorous level and an elevated alkaline phosphatase level are the hallmarks in diagnosing familial hypophosphatemic Vitamin D- resistant rickets. Serum calcium is usually normal or low normal. This disease is inherited as an X-linked dominant trait and usually presents at age 18 to 24 months. The disease results from a poorly defined problem with renal phosphate transport in which normal dietary intake of vitamin D is insufficient to achieve normal bone mineralization. Renal tubular dysfunction is associated with urinary phosphate wasting. Treatment involves oral phosphate supplementation, which can cause hypocalcemia and secondary
hyperparathyroidism. To prevent associated problems, high doses of Vitamin D are administered. While obtaining a scanogram may be clinically indicated in
an associated limb-length discrepancy, and subsequent corrective surgery may be indicated, either of these choices would not be the first course of action. An orthosis may slow the progression of genu varum in this disorder but is less important than establishing the correct diagnosis to begin pharmacologic treatment. This amount of varum and tibial bowing far exceeds the normal limits of physiologic genu varum. Skeletal dysplasias usually are not
associated with abnormal laboratory values.
-
What is the peak period of onset in children with pauciarticular juvenile rheumatoid arthritis?
-
Before age 2 years
-
Between the ages of 2 and 4 years
-
Between the ages of 4 and 8 years
-
Between the ages of 8 and 12 years
-
During adolescence
CORRECT ANSWER: 2
Approximately one half of patients with juvenile rheumatoid arthritis (JRA) have the pauciarticular form, which by definition includes only patients with fewer than five joints involved. The peak period of onset is between the ages
of 2 and 4 years, with half of the affected children coming to medical attention before age 4 years. The knee is most often affected, with the ankle-subtalar and elbow joints next in frequency. The average duration of the disease is 2 years and 9 months, with half the cases lasting less than 2 years.
-
Osteonecrosis of the jaw has been recognized as a possible complication of chronic therapy with which of the following medications?
-
Warfarin
-
Low-molecular weight heparin
-
Diclofenac
-
Ketorolac
-
Alendronate
CORRECT ANSWER: 5
Migliorati et al review bisphosphonate-associated osteonecrosis. They state that it is characterized by the unexpected appearance of necrotic bone in the oral cavity.
Risk factors include intravenous therapy, duration of therapy >36mo, elderly patients with multiple myeloma, and those with a history of recent dental extraction. Incidence ranges from 4-17%.
Van den Wyngaert et al confirm the predisposition of multiple myeloma patients to develop osteonecrosis of the jaw, and conclude that although various treatment strategies have been reported, conservative management remains the mainstay of therapy.
Illustration A shows exposed bone in the mouth of a patient with osteonecrosis of the jaw who was on long term bisphosphonate therapy.
-
A 3-year-old child has bilateral genu varum and short stature. Radiographs show physeal widening and generalized osteopenia. The femora and tibiae show anterolateral bowing. Laboratory studies show low normal serum calcium values, significantly decreased serum phosphate levels, elevated alkaline phosphatase levels, and normal parathyroid hormone (PTH),and vitamin-D levels. These findings are most consistent with:
-
nutritional rickets.
-
renal osteodystrophy.
-
primary hyperparathyroidism.
-
hypophosphatasia.
-
vitamin D-resistant rickets.
CORRECT ANSWER: 5
The clinical condition is most consistent with vitamin D-resistant rickets.
Children with vitamin D-resistant rickets are short in stature and have genu varum, physeal widening, and generalized osteopenia. The abnormality in inherited vitamin D- resistant rickets is the renal tubule’s inability to resorb phosphate leading to hypophosphatemia. Laboratory findings in the condition are normal or near normal serum calcium values, significantly decreased
serum phosphate levels, elevated alkaline phosphatase levels, and normal PTH and vitamin-D levels. The most common form is inherited as an X-linked dominant trait.
Illustration A is chart that shows the lab values of different types of Rickets. Incorrect Answers:
Answer 1: Nutritional rickets has a normal or low serum phosphate levels, normal or low serum calcium values, and decreased levels of 25(OH) vitamin D and 1,25- dihydroxyvitamin D.
Answer 2: Children with renal osteodystrophy tend to have genu valgum, and laboratory findings include elevated serum phosphate, alkaline phosphatase, and PTH levels, and low serum calcium values. Findings of renal disease include elevated BUN and creatinine.
Answer 3: Primary hyperparathyroidism usually is caused by a parathyroid adenoma,
and the child generally has abdominal problems and hypercalcemic crisis. Laboratory findings include elevated serum calcium values, alkaline phosphatase levels, and PTH levels, and decreased serum phosphate levels. Answer 4: Hypophosphatasia is a rare condition characterized by a deficiency of alkaline phosphatase in the serum and tissues, leading to generalized
abnormal mineralization of bone.
-
What is the primary indication for performing an arthroscopic synovectomy on a patient with hemophilia that is the result of factor VIII deficiency?
-
Joint pain with radiographic evidence of joint space narrowing
-
Joint stiffness that has not improved with physiotherapy and bracing
-
Recurrent joint bleeding despite optimal medical management
-
Prophylaxis for joint preservation in severe hemophilia (factor VIII level
<1%)
-
Decreasing ambulatory endurance despite optimal medical management Correct answer: 3
Improved medical management has changed musculoskeletal outcomes for individuals with hemophilia. Patients with severe hemophilia receiving prophylactic administration of factor VIII may never develop a target joint that requires further orthopaedic intervention. Patients with moderate hemophilia and those patients with severe hemophilia not receiving prophylactic
treatment will still develop joints that have recurrent hemarthroses. When recurrent hemarthrosis continues despite optimal medical management,
synovectomy is indicated. While synovectomy is predictable in its ability to decrease joint bleeding, it does not necessarily improve joint range of motion or prevent the development of hemophilic arthropathy over time. It will not reverse articular damage to the joint once it has developed.
-
The husband of a 22-year-old woman has hypophosphatemic rickets. The woman has no orthopaedic abnormalities, but she is concerned about her chances of having a child with the same disease. What should they be told regarding this disorder?
-
Their sons will have a 50% chance of having this X-linked dominant disorder.
-
All of their daughters will be carriers or will have this disorder.
-
They should be advised to not have any children as the risk of having boys with the
disorder and girls who will be carriers is too hard for any parent.
-
As long as the woman does not carry the trait, the children will not be affected because the husband has the disease and this is an X-linked dominant disorder.
-
Their sons or daughters may be born with this disorder, but males are more severely affected.
CORRECT ANSWER: 2
Hypophosphatemia is a rare genetic disease usually inherited as an X-linked dominant trait. The fact that the woman has no skeletal manifestations would indicate that the husband has the X-linked mutation. The disease is more severe in boys than it is in girls. The husband will not transmit the disease to his sons. However, all of their daughters will be affected either with the disease or as carriers. If the woman has the disease or the trait, there is a
50% chance that her sons will inherit the disease and a 50% chance that her daughters will be carriers or have a milder form of the disease. Parents should be advised to have genetic counseling so they can be informed when deciding whether to have children.
-
Figure 9 shows the AP radiograph of a 65-year-old man who has knee pain and swelling. What is the most likely diagnosis?
-
Gout
-
Chondrocalcinosis (pseudogout)
-
Hemochromatosis
-
Rheumatoid arthritis
-
Ochronosis
CORRECT ANSWER: 2
Although all the choices are known causes of joint degeneration (secondary osteoarthritis), only chondrocalcinosis shows distinct linear calcification of the cartilage due to deposition of calcium pyrophosphate crystals. Gout is a recurrent acute arthritis resulting from the deposition of monosodium urate from supersaturated hyperuricemic body fluids. Hemochromotosis is characterized by focal or generalized deposition of iron within body tissues. Arthritis may be present but is less common than other manifestations such as liver cirrhosis, skin pigmentation, diabetes mellitus, and cardiac disease.
Rheumatoid arthritis is a nonspecific, usually symmetric inflammation of peripheral joints resulting in destruction of articular and periarticular structures. Ochronosis is a hereditary enzyme deficiency (homogentisic acid oxidase) resulting in deposition of homogentisic acid polymers in articular cartilage.
-
A 48-year-old woman reports bilateral thigh pain that is limiting her function as a librarian. A radiograph and a bone scan are shown in Figures 23a and 23b. What is the most likely diagnosis?
-
Ankylosing spondylitis
-
Arthrokatadysis
-
Osteomalacia
-
Rheumatoid arthritis
-
Developmental dysplasia
CORRECT ANSWER: 4
The radiograph reveals bilateral severe acetabular protrusio. The bone scan and history confirm involvement of multiple joints, including the knees and the hindfoot. Although the first four choices can all cause the acetabular protrusio, the associated multiple joint involvement suggests the diagnosis of rheumatoid arthritis. Arthrokatadysis, or primary protrusio acetabuli, is often associated with osteomalacia but not other joint disease.
Developmental dysplasia is a common cause of bilateral hip pathology but does not have acetabular protrusio.
-
A 25-year-old man sustains a Grade III-A open tibial shaft fracture secondary to a motorcycle accident. The patient is
unconscious and has no family members who can be reached for consent. What is the most appropriate course of action?
-
Document the necessity for treatment and proceed immediately with definitive fracture care
-
Proceed with a preliminary irrigation and debridement in the emergency department, apply a splint, and wait for him to regain consciousness before proceeding with definitive treatment
-
Contact a hospital administrator for approval of care
-
Confirm and document the necessity of care with a colleague with similar expertise and knowledge prior to proceeding with surgery
-
Proceed immediately to the operating room for definitive treatment without further documentation
CORRECT ANSWER: 4
A legal guardian should provide informed consent in the case where a patient does not have capacity and needs emergency surgery. Next in chain of command is a family member if no legal guardian exists. Lastly, if neither a guardian or family member is available, it is appropriate to have another physician, with similar expertise and knowledge as the proposed surgeon, confirm the necessity of the procedure. This should be well-documented prior to proceeding with the procedure.
The article by Naarden and Cissik reviews strategies to improve the informed consent process.
-
Figure 35 shows the radiograph of a 44-year-old woman with rheumatoid arthritis who reports neck pain. Below what threshold number is surgical stabilization warranted for the interval shown by the arrow?
-
8 mm
-
10 mm
-
12 mm
-
14 mm
-
16 mm
CORRECT ANSWER: 4
The posterior atlanto-dens interval represents the space available for the spinal cord and a distance of less than 14 mm is predictive of neurologic progression, thus warranting consideration for fusion, even in the absence of symptoms.